You are on page 1of 379

KARNATAKA STATE OPEN UNIVERSITY

MANASAGANGOTRI, MYSORE-570006

MCA 11
MATHEMATICS
BLOCK 1 ALGEBRAIC STRUCTURES

In collaboration with

VIRTUAL EDUCATION TRUST


Registered Office
UU -11, Pitampura, Delhi 110 034 Ph : 011-27348396 / 27348107 / 27345121 Fax : 011-27345042 E : vet@pctiltd.com, W : http://www.vet.pctiltd.com

First Edition 2009

Copyright by Virtual Education Trust All right reserved

No part of this publication may be reproduced in any form or by any means without the written permission from the Virtual Education Trust

All product names and company names mentioned

herein are the property of their respective owner.

Not for sale For personaluse of KSOU Informationtechnology&Management programme student only

Course Introduction

Engineering physics is a branch of applied science that emphasizes both engineering and physics. The engineering physics curriculum is designed to fulfill the educational requirements for professional work in various fields of applied science which are based upon a thorough knowledge of physics and foundation of basic scientific principles, as well as the theoretical knowledge and skills required for specific engineering applications.

The study of Engineering Physics emphasizes the application of basic scientific principles to the design of equipment, which includes electronic and electromechanical systems, for use in measurements, communications, and data acquisition.

The course is recommended for students interested in newly developing areas of physics, high technology, instrumentation and communications.

Subject Introduction

This block explains Trigonometric Functions, Periodic Functions , Evaluation of Trigonometric functions , The Magic Identity, The Addition , Formulas, DoubleAngle and Half-Angle Formulas and Cramers Rule, Eigenvalues, Eigenvectors, and Definiteness, Matrix Diagonalization. Unit 1 Objectives , Introduction , Trigonometric Functions, Periodic Functions , Evaluation of Trigonometric functions , The Magic Identity, The Addition , Formulas, Double-Angle and Half-Angle Formulas, Product and Sum Formulas, Complex Number, Properties of Complex Numbers, Polar form the Complex Number, DeMoivre's Theorem, Roots of a Complex Number, Summary, Keywords, Self Assessment Questions, References. Unit 2 Objectives, Introduction, Matrix Operations, Addition, Multiplication by a number, Multiplication, Matrix and System of Linear Equations, Determinant and Inverse of Matrix, Cramers Rule, Eigenvalues, Eigenvectors, and Definiteness, Matrix Diagonalization, The CayleyHamilton Theorem, Summary, Keywords, Self Assessment Questions, References. Unit 3 Objectives, Introduction, Definition of a group, Examples of Groups, Properties of Groups, Uniqueness of Identity Element, Uniqueness of inverse, Cancellation laws, Notation, Order, Symmetric Groups,

Subgroups, Subgroups and subgroup tests, Cyclic groups, Cosets, Scalar and Vector Products ,Vectors, Vector Addition, Subtraction, Unit Vector, Summary, Keywords, Self Assessment Questions, References. Unit 4 Objectives, Introduction, Definition of Limits, A Function with a jump, Another type of jump: the Heaviside function, Basic Properties of Limits,

Continuity, References.

Summary,

Keywords,

Self

Assessment

Questions,

Mathematics Algebraic Structures

Unit 1 Complex Trigonometry 4

Unit 2 Matrix Theory 38

Unit 3 Algebraic Structures 61

Unit 4 Limits and Continuity 83

Unit 1 Complex Trigonometry


Structure

1.0 1.1 1.2 1.3 1.4 1.5 1.6 1.7 1.8 1.9 1.10 1.11 1.12 1.13 1.14 1.15

Objectives Introduction Trigonometric Functions Periodic Functions Evaluation of Trigonometric functions The Magic Identity The Addition Formulas Double-Angle and Half-Angle Formulas Product and Sum Formulas Complex Number Properties of Complex Numbers Polar form the Complex Number DeMoivre's Theorem Roots of a Complex Number Summary Keywords

1.16 1.17

Self Assessment Questions References

1.0

Objectives
After studying this unit you will be able to: Define Trigonometric Functions Explain Periodic Functions Evaluate Trigonometric functions Describe the Magic Identity Describe Double-Angle and Half-Angle Formulas Discuss Product and Sum Formulas Define Complex Number

1.1

Introduction
Let us understand that any real number measure of an angle as follows: If may be interpreted as the radian

, think of wrapping a length of string

around the standard unit circle C in the plane, with initial point P(1,0), and proceeding counterclockwise around the circle; do the same if , but wrap

the string clockwise around the circle. This process is described in Figure 1 below.

If Q(x,y) is the point on the circle where the string ends, we may think of as being an angle by associating to it the central angle with vertex O(0,0) and sides passing through the points P and Q. If instead of wrapping a length s of string around the unit circle, we decide to wrap it around a circle of radius R, the angle (in radians) generated in the process will satisfy the following relation:

Observe that the length s of string gives the measure of the angle only when R=1.

As a matter of common practice and convenience, it is useful to measure angles in degrees, which are defined by partitioning one whole revolution into 360 equal parts, each of which is then called one degree. In this way, one whole revolution around the unit circle measures is: radians and also 360 degrees (or ), that

Each degree may be further subdivided into 60 parts, called minutes, and in turn each minute may be subdivided into another 60 parts, called seconds:

EXAMPLE 1 Express the angle notation.

in Degree-Minute-Second (DMS)

Solution: We use Equation 3 to convert a fraction of a degree into minutes and a fraction of a minute into seconds:

Therefore,

EXAMPLE 2 Express the angle

in radians.

Solution: From Equation 2 we see that

EXAMPLE 3 Find the length of an arc on a circle of radius 75 inches that spans a central angle of measure .

Solution:

We

use

Equation 1,

with

R=75

inches

and

, to obtain

Here are some more exercises in the use of the rules given in Equations 1,2, and 3.

1.2

Trigonometric Functions
A real number can be interpreted as the measure of the angle constructed as follows: wrap a piece of string of length units around the unit circle (counterclockwise if , clockwise if ) with initial point P(1,0) and

terminal point Q(x,y). This gives rise to the central angle with vertex O(0,0) and sides through the points P and Q. All six trigonometric functions of are defined in terms of the coordinates of the point Q(x,y), as follows:

Since Q(x,y) is a point on the unit circle, we know that

. This fact

and the definitions of the trigonometric functions give rise to the following fundamental identities:

This modern notation for trigonometric functions is due to L. Euler (1748).

More generally, if Q(x,y) is the point where the circle

of radius R is

intersected by the angle , then it follows (from similar triangles) that

1.3

Periodic Functions
If an angle corresponds to a point Q(x,y) on the unit circle, it is not hard to see that the angle corresponds to the same point Q(x,y), and hence that

Moreover,

is the smallest positive angle for which Equations 1 are true for any

angle . In general, we have for all angles :

We call the number

the period of the trigonometric functions and

and

, and

refer to these functions as being periodic. Both as well, with period , while and

are periodic functions

are periodic with period .

EXAMPLE 1 Find the period of the function

Solution: The function angle 3x runs from 0 to period of f(x) is then .

runs through a full cycle when the , or equivalently when x goes from 0 to . The

1.4

Evaluation of Trigonometric functions

Consider the triangle with sides of length Figure 1 below:

and hypotenuse c>0 as in

Figure 1

For the angle pictured in the figure, we see that

There are a few angles for which all trigonometric functions may be found using the triangles shown in the following Figure 2.

This list may be extended with the use of reference angles (see Example 2 below).

EXAMPLE 1: Find the values of all trigonometric functions of the angle

Solution: From Figure 2, we see that the angle of on the unit circle, and so

corresponds to the point

EXAMPLE 2: Find the values of all trigonometric functions of the angle .

Solution: Observe that an angle of total of ) plus

is equivalent to 8 whole revolutions (a and intersect the unit

, Hence the angles

circle at the same point Q(x,y), and so their trigonometric functions are the same. Furthermore, the angle of makes an angle of with respect to the x-axis

(in the second quadrant). From this we can see that and hence that

We call the auxiliary angle of

the reference angle of

EXAMPLE 3 Find all trigonometric functions of an angle in the third quadrant for which .

Solution: We first construct a point R(x,y) on the terminal side of the angle , in the third quadrant. If R(x,y) is such a point, then see that we may take x=-5 and R=6. Since and we we find that

(the negative signs on x and y are taken so that R(x,y) is a point on the third quadrant, see Figure 3).

It follows that

1.5

The Magic Identity

Trigonometry is the art of doing algebra over the circle. So it is a mixture of algebra and geometry. The sine and cosine functions are just the coordinates of a point on the unit circle. This implies the most fundamental formula in trigonometry (which we will call here the magic identity)

where is any real number (of course measures an angle).

Example. Show that

Answer. By definitions of the trigonometric functions we have

Hence we have

Using the magic identity we get

This completes our proof.

Remark. the above formula is fundamental in many ways. For example, it is very useful in techniques of integration.

Example. Simplify the expression

Answer. We have by definition of the trigonometric functions

Hence

Using the magic identity we get

Putting stuff together we get

This gives

Using the magic identity we get

Therefore we have

1.6

The Addition Formulas


The fundamental identities are very important for the analysis of trigonometric expressions and functions but they are a direct result of the intimate relation between trigonometry and geometry. The power behind the algebraic nature of trigonometry is hidden and can be measured only with the addition formulas

and

Of course, we used the fact that

Example. verify the identity

Answer. We have

which gives

But

and since

and

, we get finally

Remark. In general it is good to check whether the given formula is correct. One way to do that is to substitute some numbers for the variables. For example, if we take a=b = 0, we get

or we may take

. In this case we have

Example. Find the exact value of

Answer. We have

Hence, using the additions formulas for the cosine function we get

Since

we get

Example. Find the exact value for

Answer. We have

Since

we get

Finally we have

Remark. Using the addition formulas, we generate the following identities

More identities may be proved similar to the above ones. The bottom line is to remember the addition formulas and use them whenever needed.

1.7

Double-Angle and Half-Angle Formulas


Double-Angle and Half-Angle formulas are very useful. For example, rational functions of sine and cosine wil be very hard to integrate without these formulas. They are as follow

Example. Check the identities

Answer. We will check the first one. the second one is left to the reader as an exercise. We have

Hence

which implies

Many functions involving powers of sine and cosine are hard to integrate. The use of Double-Angle formulas help reduce the degree of difficulty.

Example. Write with their first power.

as an expression involving the trigonometric functions

Answer. We have

Hence

Since

, we get

or

Example. Verify the identity

Answer.We have

Using the Double-Angle formulas we get

Putting stuff together we get

From the Double-Angle formulas, one may generate easily the Half-Angle formulas

In particular, we have

Example. Use the Half-Angle formulas to find

Answer. Set

. Then

Using the above formulas, we get

Since

, then

is a positive number. Therefore, we have

Same arguments lead to

Example. Check the identities

Answer. First note that

which falls from the identity one identity. For example, let us verify that

. So we need to verify only

using the Half-Angle formulas, we get

which reduces to

1.8

Product and Sum Formulas


From the Addition Formulas, we derive the following trigonometric formulas (or identities)

Remark. It is clear that the third formula and the fourth are identical (use the property to see it).

The above formulas are important whenever need rises to transform the product of sine and cosine into a sum. This is a very useful idea in techniques of integration.

Example. Express the product functions.

as a sum of trigonometric

Answer. We have

which gives

Note that the above formulas may be used to transform a sum into a product via the identities

Example. Express

as a product.

Answer. We have

Note that we used

Example. Verify the formula

Answer. We have

and

Hence

which clearly implies

Example. Find the real number x such that

and

Answer. Many ways may be used to tackle this problem. Let us use the above formulas. We have

Hence

Since

, the equation

gives

and the equation

gives

. Therefore, the solutions to the equation

are

Example. Verify the identity

Answer. We have

Using the above formulas we get

Hence

which implies

Since

, we get

1.9

Complex Number
Definition : A number of the form x + iy where is defined

as a Complex Number and usually denoted as Z. x is called Real part & y is called Imaginary part. x - iy is called Conjugate. Complex number denoted as . A complex number can be represented by a point on a plane by taking real part on x-axis & imaginary part on y-axis. The plane on which complex numbers are represented is called a Complex Plane. For every point in a plane there is a complex number & for every complex number there is a point in the plane. In a complex x-axis is called Real axis & y-axis Imaginary axis.

1.10 Properties of Complex Numbers

Equality. Two complex numbers

are said to be equal if

Addition. If

Subtraction.

Multiplication.

(5) Division.

multiply numerator & denominator by the conjugate of

, then

which is a complex number.

Note :- Product of a complex number with its conjugate is always a positive real

number.

1.11 Polar form the Complex Number


z = x + iy is called the Cartesian form.

Let P(x, y) be any point in the plane which represents a complex number. Draw to x-axis & join PM. Then OM = x, MP = y. Let & OP = r.

From the triangle OPM,

This form of the complex number is called 'Polar form'. Where r is called Modulus & q is called argument which are given by

r is always positive and argument q varies from 0 to 360. The value of argument satisfying number. is defined as amplitude which is unique for a complex

Thus we have

while finding the amplitude of a complex number, we have to find q satisfying ie

Find the modulus & amplitude of

Solution :

1.12 DeMoivre's Theorem


Statement : If n is a +ve or ve integer, then

If

is

+ve

or

ve

fraction,

one

of

the

values

of

Proof : Case (i) when n is a +ve integer proof by Mathematical Induction. When n = 1,

therefore result is true for n = 1.

Let us assume that the result is true for n = m

the result is true for n = m + 1. Thus if the result is true for n = m then it is true for n = m + 1. ie If it is true for one integer it is true for next integer, hence by Induction the result is true for all +ve integers.

Case (ii) When

where p & q are +ve integers.

taking qth roots on both sides. One of the values of

Case (iii) when n is -ve integer or -ve fraction.

Let

-m

where

is

+ve

integer

or

+ve

fraction

Important Results

Note :- For convenience

can be written as

1.13 Roots of a Complex Number


Let z = x + iy, express the complex number in the polar form.

where k = 0, 1, 2,

n - 1. Let us denote the nth roots of the complex number by

Then,

the above n values gives nth roots of z = x + iy

Note :- If k = n, n + 1, n + 2 etc. The values will repeat. Hence these will be only n values of which are distinct. Using the polar form of the complex number we

can plot the nth roots of the complex in the following way.

Draw a circle of radius r 1 n whose centre is O. Mark a point on the circle and takeOA as intial line. Take a point B such that AOB = . Then B represent z. Take

a point C such that

then C represent z1, like this all the nth

roots can be represented. This diagram is called 'Argand Diagram'

Problems :

Similarly if

Adding (1) and (2)

1.14 Summary
As a matter of common practice and convenience, it is useful to measure angles in degrees, which are defined by partitioning one whole revolution into 360 equal parts, each of which is then called one degree. Trigonometry is the art of doing algebra over the circle. So it is a mixture of algebra and geometry. The sine and cosine functions are just the coordinates of a point on the unit circle.

1.15 Keywords
The Magic Identity: Trigonometry is the art of doing algebra over the circle. So it is a mixture of algebra and geometry. The sine and cosine functions are just the coordinates of a point on the unit circle. This implies the most fundamental formula in trigonometry (which we will call here the magic identity)

1.16 Self Assessment Questions


1. Define Trigonometric Functions 2. Explain Periodic Functions 3. Evaluate Trigonometric functions 4. Describe the Magic Identity 5. Describe Double-Angle and Half-Angle Formulas 6. Discuss Product and Sum Formulas 7. Define Complex Number

1.17 References
Elementary Engineering Mathematics by Dr. B.S. Grewal, Khanna Publications. Higher Engineering Mathematics by B.S. Grewal, Khanna Publications. Differential Calculus by Shanti Narayan, Publishers S. Chand & Co. Integral Calculus by Shanti Narayan, Publishers S. Chand & Co.

Unit 2 Matrix Theory


Structure

2.0 2.1 2.2

Objectives Introduction Matrix Operations 2.2.1 Addition 2.2.2 Multiplication by a number 2.2.3 Multiplication

2.4 2.5 2.6 2.7 2.8 2.9 2.10 2.11 2.12 2.13

Matrix and System of Linear Equations Determinant and Inverse of Matrix Cramers Rule Eigenvalues, Eigenvectors, and Definiteness Matrix Diagonalization The Cayley-Hamilton Theorem Summary Keywords Self Assessment Questions References

2.0

Objectives
After studying this unit you will be able to: Explain Matrix Operations Discuss Matrix and System of Linear Equations Describe Determinant and Inverse of Matrix Define Cramers Rule Evaluate Eigenvalues, Eigenvectors, and Definiteness Discuss Matrix Diagonalization Explain The Cayley-Hamilton Theorem

2.1

Introduction
Let us understand that Now well start studying new algebraic object matrices.

Definition 1.1. The matrix is a (rectangular) table of the elements ofR. (Actually, we can consider matrices over fields other than R - in the future we will work with matrices over the field of complex numbers C.)

Now well introduce some notation that we will use. We will denote matrices with capital letters, and the elements of the matrix with same small letter with 2 subscripts, the first of them denotes the row, and the second one denotes the

column. Often we will speak about m x n-matrices, which means that it has m rows and n columns.

The matrix is called square matrix if the number of its rows is equal to the number of its columns. For every square matrix we will define its main diagonal, or simply diagonal, as a diagonal from the top left corner to the bottom right corner, i.e. diagonal consists of the elements a11, a22, . . ., ann. Another diagonal is called secondary. It is used very rarely.

So, we introduced an object. But now we should introduce operations, otherwise the object is not interesting!

2.2

Matrix Operations
2.2.1 Addition The first and the easiest matrix operation is matrix addition. Definition 2.1. Let A and B are m n-matrices. Then their sum C = A + B is an m n-matrix such that i.e. the elements of this matrix are sums of

corresponding elements of the matrices A and B.

Example 2.2.

Now lets consider the properties of addition.

(A1) Commutativity. It is obvious that for any matrices A and B of the same size A+B = B + A.

(A2) Associativity. It is obvious that for any matrices A, B and C of the same size (A + B) + C = A + (B + C).

Here we can mention that we can choose any order of matrices to perform the addition of 3 or more matrices. For example, we can prove, that (A + B) + C = (A + C) + B.

P roof.

(A+C)+ B = (C +A)+B = C +(A+B) = (A+B)+C

(A3) Existence of the zero. There exists a zero matrix

For any matrix A we have that A + 0 = A.

(A4) Existence of the additive inverse. For any matrix A there exists matrix -A such that A + (-A) = 0. The elements of this matrix (-a)ij = -aij.

Example 2.3. The additive inverse for the matrix

2.2.2 Multiplication by a number For any matrix A and for any number c G R we can define the matrix B = cA, such that bij = caij, i.e. we multiply all elements of the matrix A by the same number c. This operation has the following obvious properties:

2.2.3 Multiplication The definition of multiplication is much more complicated than the definition of the previous operations.

Definition 2.4. Let A be an m x p-matrix and B be a p x n-matrix. Then their product is an m x n matrix C such that

So, we see, that in order to be able to multiply matrices, the number of columns of the first matrix should be equal to the number of rows of the second one.

Example 2.5.

We can see, that we cannot multiply these 2 matrices in different order, i.e. we can not compute

Example of matrix multiplication. Let and be real numbers, and lets compute the following product

It is equal to:

So,we can see that we get a matrix of the same type, but instead of and we have + .

Now lets consider the properties of multiplication.

Commutativity.

Unfortunately,

commutativity

does

not

hold

for

matrix

multiplication. Moreover, for some matrices A and B we can compute AB and cannot compute BA. E.g., if A is a 2 3-matrix, and B is a 3 3-matrix, then AB is defined, and BA is not. Now, we can give a counterexample even if both products are defined.

Moreover, from this example we see that the product of two nonzero matrices can be a zero matrix.

(M2) Associativity. Associativity holds for matrix multiplication, i.e. for any three matrices such that all needed products(i.e., AB and BC) can be defined, we have that (AB)C = A(BC).

Proof. Let

Then

and

In the same way

and so

Now we can change the order of the summation, and see that these expressions are equal.

Here, unlike in the case of addition, we cannot choose any order, since commutativity does not hold for multiplication. For example,

(AB)C = (CA)B, etc.

The order of the matrices one multiplies should be always preserved.

(M3) Existence of the one. There exists an identity matrix

This n n-matrix has 1s on its main diagonal. For any m n-matrix A we have that AI = IA = A.

Proof. Can be done directly from the definition of the matrix multiplication. Simply can check that

By the same arguments, IA = A So, this matrix plays the same role for matrices as a number 1 for numbers. By multiplying by identity matrix, we do not change the given matrix.

2.4

Matrix and System of Linear Equations


Definition Let A and B be m x n matrices.

Example Calculate

and BA using the following matrices.

The basic algebraic operations for matrices are as follows:

Theorem 1 (Algebraic Operations of Matrices) Let A, B, C be matrices of appropriate sizes. 1. Addition:

A + B = B + A and A + (B + C) = (A + B) + C There exists a unique C such that A + C = A and we denote C = O.

There exists a unique C such that A + C = O and we denote C = -A.

2.

Multiplication:

(a)

(b) A(BC) = (AB)C.

(c) (A + B)C = AC + BC and C(A + B) = CA + CB.

3. Transpose:

Definition Let A be an m x n matrix. A is called a square matrix if n = m. A is called symmetric if A = A.

A square matrix A is called a diagonal matrix ifaij = 0 for i = j. A is called upper triangular if aij = 0 for i > j and called lower triangular if aij = 0 for i < j. A diagonal matrix A is called an identity matrix if aij = 1 for i = j and is denoted by In.

In particular, we have

for any square matrix A. For a square

matrix, there is another operator called trace.

Theorem (Trace Operator) Let A and B be matrices of appropriate sizes.

If we start out with an m x n matrix and delete some, but not all, of its rows or columns, then we obtain a submatrix.

A matrix can be partitioned into submatrices, and such a matrix is called partitioned matrices. Partitioned matrices can be manipulated in the same way (called block manipulation) provided that submatrices are of appropriate sizes.

Definition An n xn matrix A is called nonsingular or invertible if there exists ann xn matrix B such that AB = BA = In We call such B an inverse of A. Otherwise, A is called singular or noninvertible.

Theorem (Uniqueness of Inverse) The inverse of a matrix, if it exists, is unique. We denote the unique inverse of A by A-1.

Theorem (Properties of Inverse) Let A and B be nonsingular n xn matrices. AB is nonsingular and (AB)-1 = B-1A-1. A-1 is nonsingular and (A-1)-1 = A.

One application of inverting a matrix is to solve a system of linear equations. In fact, matrices can be motivated in terms of linear equations. Consider a set of m linear equations of the form

Then, its matrix representation is Y = AX where

We call A a coefficient matrix. With this notation, we can see that that A is nonsingular) solves this system since we obtain X = premultiplying the equation by .

(provided Y by

Definition Elementary row (column) operations on an m n matrix A includes the following.

Interchange rows (columns) r and s of A. Multiply row (column) r of A by a nonzero scalar k = 0. Add k times row (column) r of A to row (column) s of A where r = s.

An mn matrix A is said to be row (column) equivalent to an mn matrix B if B can be obtained by applying a finite sequence of elementary row (column) operations to A.

Definition An m n matrix A is said to be in reduced row echelon form if it satisfies the following properties.

All rows consisting entirely of zeros, if any, are at the bottom of the matrix. By reading from left to right, the first nonzero entry in each row that does not consist entirely of zeros is a 1, called the leading entry of its row.

If rows i and i + 1 are two successive rows that do not consist entirely of zeros, then the leading entry of row i + 1 is to the right of the leading entry of row i. If a column contains a leading entry of some row, then all other entries in that column are zero.

If A satisfies 1, 2, and 3, but not 4, then it is said to be in row echelon form. A similar definition can be applied to (reduced) column echelon form.

2.5

Determinant and Inverse of Matrix


In this section, we will learn how to find the inverse of a matrix.

Definition: A permutation of a finite set of integers S = {1,2,... ,n} is a bijective function . A permutation is said to have an inversion if a larger

integer precedes a smaller one. A permutation is called even (odd) if the total number of inversions is even (odd).

That is, if S = {1,2,3}, then f defined by f(1) = 3,f(2) = 2,f(3) = 1 is an odd permutation. Now, we are ready to define determinant of a matrix.

Definition : Let A be an n x n matrix. Then, the determinant of A denoted by or det(A) is permutations where the summation is over all with S = {1,2,..., n}. The sign is + () if the

corresponding permutation is even (odd).

Now, we compute the determinants of the following matrices. It should be noted that there is no easy method for computing determinants for n > 3.

Example What are the determinants of 1 x 1, 2 x 2, and 3x3 matrices? We examine some basic properties of determinants. In particular, there is an important relationship between the singularity and the determinant of a matrix.

Theorem (Determinants) Let A and B be n x n matrices.

According to Definition, computing the determinant of an n x n matrix can be very cumbersome if n is large. We now develop a method which reduces the problem to the computation of the determinant of an (n 1) x (n 1) matrix so that we can repeat the process until we get to a 2 x 2 matrix.

Definition : Let A be an n x n matrix.

Let Mij be the (n 1) x (n 1) submatrix of A obtained by deleting the ith row and jth column of A. Then, |Mij| is called the minor of aij.

The cofactor of Aij of aij is defined as

Now, the following theorem gives us a new method to compute determinants.

Definition: Let A be an n n matrix. The adjoint of A, adjA, is the matrix whose (i, j) element is the cofactor That is,

2.6

Cramers Rule
Theorem (Cramers Rule) Consider a system of n linear equations in n unknown parameters with the coefficient matrix A so that we can write Y = AX

where Ai is the matrix obtained from A by replacing its ith column by Y .

Example Apply the Cramers Rule to the following system of linear equations.

2.7

Eigenvalues, Eigenvectors, and Definiteness


As the final topic of this chapter, we study eigenvalues and eigenvectors. Although we will not prove many of the theorems, their results are important and will be frequently applied in statistics. First, we give the definition of eigenvalues and eigenvectors.

Definition Let A be ann xn square matrix. G R is called an eigenvalue of A if there exists a nonzero vector x such that Ax = x. Every nonzero vector satisfying this equation is called an eigenvector of A associated with the eigenvalue .

Note that x = 0 always satisfies the equation, but it is not an eigenvector. Example Confirm that 1 = 2 and 2 = 3 are the eigenvalues and x1 = (1,1) and x2 = are their associated eigenvectors of

The connection between eigenvalues and singularity is critical.

Theorem (Eigenvalues) Let A be an n xn square matrix and 1,..., n be its eigenvalues.

If A is diagonal, then its diagonal elements are the eigenvalues.

In particular, A is singular if and only if 0 is an eigenvalue of A. Before we state the key theorem, we need one more concept.

Definition Let A be an n x n square matrix. Then, characteristic polynomial of A. The equation characteristic equation of A.

is called the = 0 is called the

The next theorem shows how one can find eigenvalues.

Theorem (Characteristic Polynomial) Let A be an n x n matrix. The eigenvalues of A are the real roots of the characteristic polynomial of A. A is said to be diagonalizable if all the roots of its characteristic polynomial are real and distinct.

The word diagonalizable comes from the fact that the diagonal matrix whose nonzero elements are the eigenvalues of A represent a linear transformation, a function mapping from one real vector space to another. is a linear

transformation if it satisfies L(v + w) = L(v) + L(w) and L(cv) = cL(v) for any vector

One important linear transformation is what is called projection defined by

and represented by the matrix A =

so that for

any vector

we have

Example Find the eigenvalues and eigenvectors, if they exist, of

and

Definition An n xn square matrix A is called orthogonal if

In particular, the orthogonal matrix is invertible, and hence

2.8

Matrix Diagonalization

Matrix diagonalization is the process of taking a square matrix and converting it into a special type of matrix--a so-called diagonal matrix--that shares the same fundamental properties of the underlying matrix. Matrix diagonalization is equivalent to transforming the underlying system of equations into a special set of coordinate axes in which the matrix takes this canonical form. Diagonalizing a matrix is also equivalent to finding the matrix's eigenvalues, which turn out to be precisely the entries of the diagonalized matrix. Similarly, the eigenvectors make up the new set of axes corresponding to the diagonal matrix.

The remarkable relationship between a diagonalized matrix, eigenvalues, and eigenvectors follows from the beautiful mathematical identity (the eigen decomposition) that a square matrix form can be decomposed into the very special

where

is a matrix composed of the eigenvectors of

is the diagonal matrix is the matrix inverse of

constructed from the corresponding eigenvalues, and

. According to the eigen decomposition theorem, an initial matrix equation

can always be written

(at least as long as gives

is a square matrix), and premultiplying both sides by

Since the same linear transformation

is being applied to both

and

, solving

the original system is equivalent to solving the transformed system

where

and

. This provides a way to canonicalize a system into for an

the simplest possible form, reduce the number of parameters from

arbitrary matrix to for a diagonal matrix, and obtain the characteristic properties of the initial matrix. This approach arises frequently in physics and engineering, where the technique is oft used and extremely powerful.

2.9

The Cayley-Hamilton Theorem


Some mathematical theorems are especially beautiful. It's hard to say what makes a theorem beautiful, but two properties come to mind: It's simple to state.

It's hard to prove.

In my opinion, one very beautiful theorem is the Cayley-Hamilton Theorem of matrix algebra. It states that if p(z) is the characteristic polynomial of an n x n complex matrix A, then p(A) is the zero matrix, where addition and multiplication in its evaluation are the usual matrix operations, and the constant term p 0 of p(z) is replaced by the matrix p0I.

Recall that the characteristic polynomial of A is given by

p(z) = det(A-zI).

Proof There are many proofs of the Cayley-Hamilton Theorem. I A more analytic argument, like the one presented here, is more suited to my own training and talents. It helps to work out the general 2 x 2 case:

The characteristic polynomial of A is

Substituting A for z gives:

To prove the theorem in the general case, let u be an eigenvalue of A, and let x be the corresponding eigenvector (expressed as a column vector). Then

Ax = ux.

Using elementary properties of scalar and matrix operations gives

It can be shown in general that

where A0 = I.

Let the characteristic polynomial of A be

Then multiply each equation in [1] above by pk and add them to obtain p(A)x = p(u)x.

Now p(u) is zero because u is an eigenvalue of A. Hence p(A)x = 0 for every eigenvector x of A.

If A has n linearly independent eigenvectors, this implies that p(A) must be the zero matrix.

If A does not have n linearly independent eigenvectors, we construct a sequence A1, A2, ... of matrices whose limit is A, each of which has n linearly independent eigenvectors. Then if pj(z) is the characteristic polynomial of Aj, pj(Aj) = O. Since all coefficients in pj(Aj) are continuous functions of the matrix entries, the same is true of the limit p(A).

To create such a sequence, it is sufficient to construct matrices arbitrarily close to A, each of which has n linearly independent eigenvectors.

First, we need a simple lemma. The matrix A, like all complex matrices, is similar to an upper triangular matrix, i.e., there is a nonsingular matrix Q for which

is upper triangular. This result is well-known, but a simple proof is given in Appendix A.

The eigenvalues of an upper triangular matrix appear along its principal diagonal. There is an upper triangular matrix T arbitrarily close to Q-1AQ with n distinct eigenvalues. Then QTQ-1 is arbitrarily close to A and has the same n distinct eigenvalues as T.

A matrix with n distinct eigenvalues has n distinct eigenvectors. If these eigenvectors were linearly dependent, they would span a space of dimension less than n. The mapping defined by the matrix, restricted to this space, would still have the same n distinct eigenvalues, which is impossible. Hence the eigenvectors are linearly independent.

Proof for Commutative Rings This proves the Cayley-Hamilton Theorem for complex matrices, but it is also true for matrices over more general commutative rings.

The proof of this is actually fairly simple. Our experience in proving the 2 x 2 case shows the way. The expression for each entry in p(A) is a polynomial in n2 variables, which are the entries of A. It's not just any polynomial, but one which takes on the value zero for all values of the variables. That can happen only if all the coefficients are zero when like terms are combined. (This seems to be an obvious result, but it requires proof, so one is given in Appendix B.) Hence the polynominal evaluates to zero in any other algebraic entity that has all the necessary operations.

It might appear that the ring must have a unit. However, if we refrain from combining like terms, we will have a sum of monomials, each prepended by a + sign or a - sign. Even in the complex field, cancellation is possible only if every positive monomial has a corresponding negative monomial. They will cancel in a ring, too, even if the ring has no unit.

2.10 Summary
We will denote matrices with capital letters, and the elements of the matrix with same small letter with 2 subscripts, the first of them denotes the row, and the second one denotes the column. Often we will speak about m x n-matrices, which means that it has m rows and n columns The matrix is called square matrix if the number of its rows is equal to the number of its columns. For every diagonal, as a

square matrix we will define its main diagonal, or simply

diagonal from the top left corner to the bottom right corner, i.e. diagonal consists of the elements a11, a22, . . ., ann.

2.11 Keywords
Matrix : The matrix is a (rectangular) table of the elements ofR. (Actually, we can consider matrices over fields other than R - in the future we will work with matrices over the field of complex numbers C.)

2.12 Self Assessment Questions


1. Explain Matrix Operations. 2. Discuss Matrix and System of Linear Equations 3. Describe Determinant and Inverse of Matrix. 4. Define Cramers Rule. 5. Evaluate Eigenvalues, Eigenvectors, and Definiteness. 6. Discuss Matrix Diagonalization.

7. Explain The Cayley-Hamilton Theorem.

2.13 References
Elementary Engineering Mathematics by Dr. B.S. Grewal, Khanna Publications Higher Engineering Mathematics by B.S. Grewal, Khanna Publications Differential Calculus by Shanti Narayan, Publishers S. Chand & Co. Integral Calculus by Shanti Narayan, Publishers S. Chand & Co. Modern Abstract Algebra by Shanti Narayan, Publishers S. Chand & Co.

Unit 3 Algebraic Structures


Structure

3.0 3.1 3.2

Objectives Introduction Definition of a group 3.2.1 Examples of Groups

3.3

Properties of Groups 3.3.1 Uniqueness of Identity Element 3.3.2 Uniqueness of inverse 3.3.3 Cancellation laws 3.3.4 Notation 3.3.5 Order

3.4

Symmetric Groups

3.5 Subgroups 3.5.1 Subgroups and subgroup tests 3.5.2 Cyclic groups 3.5.3 Cosets

3.6 3.7 3.8 3.9 3.10 3.11 3.12 3.13 3.14

Scalar and Vector Products Vectors Vector Addition Subtraction Unit Vector Summary Keywords Self Assessment Questions References

3.0

Objectives
After studying this unit you will be able to: Define Group Explain Examples of Groups Discuss Properties of Groups Describe Uniqueness of Identity Element Explain Symmetric Groups Elaborate Subgroups Discuss Cyclic groups Define Vectors

3.1

Introduction
Let us understand that a group is a structure with just one binary operation, satisfying four axioms. So groups are only half as complicated as rings! As well as being new material, this part will help you revise the first part of the course, since a lot of things (subgroups, homo-morphisms, Isomorphism Theorems) work in almost exactly the same way as for rings.

3.2

Definition of a group
A group is a set G with one binary operation (which we write for now as o in infix notation1) satisfying the following four axioms (G0)-(G3):

(G0) (Closure law) For any

, we have

(G1) (Associative law) For any

we have (g o h) o k = g o (h o k).

(G2) (Identity law) There is an element e EG with the property that goe = eog = g for all. (The element e is called the identity element of G.)

(G3) (Inverse law) For any element

there is an element

satisfying and call it the

e. (We denote this element h by inverse ofg.)

If a group G also satisfies the condition

Remember that this means that the result of applying the operation to a and b is written as aob.

(G4) (Commutative law) For any g,h E G, we have goh = hog,

then G is called a commutative group or (more often) an Abelian group.

3.2.1 Examples of Groups Axioms (G0)-(G4) for a group are just axioms (A0)-(A4) for a ring but using slightly different notation (the set is G instead of R, the operation is o instead of +, and so on). So we get our first class of examples:

Proposition Let R be a ring. Then R with the operation of addition is an Abelian group: the identity element is 0, and the inverse of a is a.

This group is called the additive group or R. This is not the only way to get groups from rings.

Proposition Let R be a ring with identity, and U(R) the set of units of R. Then U(R), with the operation of multiplication, is a group. If R is a commutative ring, then U (R) is an Abelian group.

This group is called the group of units of R.

Proof Let U(R) be the set of units of R.

(G0) if u and v are units, then so is uv. So U(R) is closed for multiplication.

(G1) The associative law for multiplication holds for all elements of R (by Axiom (M1) for rings), and so in particular for units.

(G2) that 1 is a unit. It clearly plays the role of the identity element.

(G3) if u is a unit, then so is

(G4) For the last part of the Proposition, if R is a commutative ring, then (M4) holds, so that uv = vu for all units. ; in particular, this holds when u and v are

3.3

Properties of Groups
Some of these properties will look very familiar, since they are similar to what we saw for rings.

3.3.1 Uniqueness of Identity Element

The identity element of a group is unique. For suppose that there are two identity elements, say e1 and e2. (This means that ge1 = e1 g = g for all g, and also ge2 = e2 g = g for all g.) Then

e1 = e1 e2 = e2.

3.3.2 Uniqueness of inverse The inverse of a group element g is unique. For suppose that h and k are both additive inverses of g. (This means that goh = hog = e and gok = kog = e - we know now that there is a unique identity element e). Then

h = hoe = ho(gok) = (hog) ok = eok = k, where we use the associative law in the third step. We denote the inverse of g by

Composing more than two elements

As long as the associative law holds, the result of composing any number of elements is independent of the way that the product is bracketed: for example, a o ((b o c) o d) = (a o b) o (c o d). Since the associative law holds in a group, we have:

Proposition Let g1,... ,gnbe elements of a group G. Then the composition

is well-defined, independent of the way it is bracketed.

3.3.3 Cancellation laws

Proposition In a group G, if then a = b.

, then a = b. Similarly, if goa = gob,

Proof Suppose that aog = bog, and let h = Then a = aoe = ao(goh) = (aog ) o h = (bog) oh = bo (goh) = boe = b.

The other law is proved similarly.

These facts are the cancellation laws.

Proposition The inverse of gh is h-1 g-1.

3.3.4 Notation

The notation things.

for the group operation is a bit cumbersome, and we now change

If we are only interested in Abelian groups, we use + as the symbol for the group operation, 0 for the group identity, and -g for the inverse of g. This agrees with the additive notation in a ring. Indeed, the additive group of a ring is an Abelian group, and every Abelian group is the additive group of a ring. [To see this, take the group operation as addition, and construct the zero ring: all products are zero.]

For general groups which may not be Abelian, we use juxtaposition for the group operation, 1 for the identity, and g-1 for the inverse of g. (This is like multiplicative notation in a ring, but it is not true that every group is the group of units in some ring!!)

This table gives the correspondences.

For the rest of this course, our notation for the group operation will be juxtaposition.

3.3.5 Order The term order has two quite different meanings in group theory: be careful not to confuse them. In the next chapter we will see that there is a close relationship between the two meanings.

The order of a group is the number of elements of the group. It may be finite (in which case it is a positive integer), or infinite. To define the second kind of order, we introduce the notation gn. This means the result of composing n factors g together:

More formally,

, and for any positive integer n,

The

order of an element g in a group is defined as follows:

If gn = 1 for some positive integer n, then the smallest such n is called the order of g.

If no such n exists, we say that g has infinite order.

Thus, the identity element always has order 1. If an element g has order 2, then it is equal to its inverse (for by the

Cancellation Law.)

Consider the additive group of the ring Z. (Recall that the operation is + and the zero element is 0; so, instead of gn we write n g, and the order is the smallest positive n such that n g = 0, or is infinite if no such n exists.) The element 1 has infinite order, since there is no positive integer n such that n 1 = 0.

In the first group in our two examples above of Cayley tables, the elements x and z have order 4 (we have x2 = y,x3 = z, x4 = e which is the identity element), while y has order 2. In the second group, all of a, b, c have order 2.

3.4

Symmetric Groups
We end this chapter by defining an important class of groups.

Let X be any set. A permutation of X is a function g : one and onto, that is, a bijection from X to X. Let Sn be the set of all permutations of the set {1,..., n}. We have

which is one-to-

For consider the two-line representation. The top row is (12 ... n). The bottom row consists of the same numbers in any order. Thus there are n possibilities for the first entry in the bottom row; n 1 possibilities for the second (anything except the first), n 2 possibilities for the third; and so on.

Now we define an operation on permutations as follows. If g is a permutation, denote the image of the element by

(As with homomorphisms, we write the function on the right of its input.) Now if g and h are two permutations, their composition g1g2 is defined by

In other words the rule is apply g, then h.

For example, if g is the permutation (1,3,5)(2,4)(6) in our above example, and h = (1,2,3,4,5,6), then gh = (1,4,3,6)(2,5). You are strongly urged to practice composing permutations given in cycle form!

Theorem The set Sn of permutations of {1,..., n}, with the operation of composition, is a group.

Proof (G0) If g and h are bijections, we have to show that gh is a bijection.

To show that it is one-to-one, suppose that x(gh) = y(gh). By definition this means (xg)h = (yg)h. Since h is one-to-one, this implies xg = yg; then, since g is one-toone, this implies x = y.

To show that it is onto, choose any element

Since h is onto, we

can find y such that yh = z. Then since g is onto, we can find x such that xg = y. Then x(gh) = (xg)h =yh = z.

(G1) Let g,h,k be three permutations. To show that g(hk) = (gh)k, we have to show that these two permutations have the same effect on any element

. Now we have

x(g(hk)) = (xg)(hk) = ((xg)h)k) = (x(gh))k = x((gh)k), as required.

(G2) The identity permutation 1 is the permutation which leaves everything as it was: that is, x1 = x for all that 1g = g; similarly g1 = g. . Then x(1g) = (x1)g = xg for all x, so

(G3) The inverse of a permutation g is simply the inverse function which undoes the effect of g: that is, if yg = x. Then it is clear that

We call this group the symmetric group on n symbols, and denote it by Sn.

3.5 Subgroups
3.5.1 Subgroups and subgroup tests A subgroup of a group G is a subset of G which is a subgroup in its own right (with the same group operation). Proposition (First Subgroup Test) A non-empty subset H of a group G is a subgroup of G if, for any

Proposition (Second Subgroup Test) A non-empty subset H of a group G is a subgroup ofG if, for any

3.5.2 Cyclic groups

If g is an element of a group G, we define the powers gn of G


1 n

as

follows: if n is positive, then gn is the product of n factors g; g0 = 1; and g-n = (g) . The usual laws of exponents hold: gm+n = gm gn and gmn = (gm)n.

A cyclic group is a group C which consists of all the powers (positive and negative) of a single element. If C consists of all the powers of g, then we write C = (g), and say that C is generated by g.

Proposition A cyclic group is Abelian. Proposition Let G be a cyclic group of finite order n. Then g has a cyclic subgroup of order m for every m which divides n; and these are all the subgroups of G 3.5.3 Cosets Given any subgroup Hofa group G, we can construct a partition of G into cosets of H, just as we did for rings. But for groups, things are a bit more complicated. Because the group operation may not be commutative, we have to define two different sorts of cosets.

Let H be a subgroup of a group G. Define a relation ~r on G by the rule x ~r y if and only if We claim that ~r is an equivalence relation:

Reflexive: For any

we have Then

Symmetric: Suppose that x ~r y, so that

Transitive: Suppose that x ~r y and y ~r z, so that h = yx-1 H. Then kh = (zy-1)(yx-1) = zx-1 H, sox ~r z.

H and k = zy-1

The equivalence classes of this equivalence relation are called the right cosets of H in G.

A right coset is a set of elements of the form Hx = {hx : h element x G called the coset representative. For

H}, for some fixed

Proposition If H is a subgroup of the group G, then G is partitioned into right cosets ofH in G, sets of the form

In a similar way, the relation

defined on G by the rule

is an equivalence relation on G, and its equivalence classes are the left cosets of H in G, the sets of the form

If G is an abelian group, the left and right cosets of any subgroup coincide, since

This is not true in general:

3.6

Scalar and Vector Products


The scalar product of the vectors a and b is

and, equivalently

Thus the scalar product is independent of the coordinate system.

The vector product is

where

is a unit vector perpendicular to both

and

. The result is invariant

with respect to the coordinate system. In cartesian coordinates it is given by

The easy way to learn this is to memorise the by cyclic rotation of the subscripts,

component and the others are obtained

The alternative method is to expand the determinant

The triple scalar product is defined as

The triple scalar product gives the volume of a parallelopiped formed by sides defined by the vectors and .

The triple vector product must be learnt. It is

Similarly,

3.7

Vectors

We will examine some of the elementary ideas concerning vectors. The reason for this introduction to vectors is that many concepts in science, for example, displacement, velocity, force, acceleration, have a size or magnitude, but also they have associated with them the idea of a direction. And it is obviously more convenient to represent both quantities by just one symbol. That is the vector.

Graphically, a vector is represented by an arrow, defining the direction, and the length of the arrow defines the vector's magnitude. This is shown in Panel 1. . If we denote one end of the arrow by the origin O and the tip of the arrow by Q. Then the vector may be represented algebraically by OQ.

This is often simplified to just

. The line and arrow above the Q are there

to indicate that the symbol represents a vector. Another notation is boldface type as: Q.

Note, that since a direction is implied,

. Even though their lengths are

identical, their directions are exactly opposite, in fact OQ = -QO.

The magnitude of a vector is denoted by absolute value signs around the vector symbol: magnitude of Q = |Q|.

The operation of addition, subtraction and multiplication of ordinary algebra can be extended to vectors with some new definitions and a few new rules. There are two fundamental definitions.

#1 Two vectors, A and B are equal if they have the same magnitude and direction, regardless of whether they have the same initial points, as shown in Panel 2.

#2 A vector having the same magnitude as A but in the opposite direction to A is denoted by -A , as shown in Panel 3.

3.8

Vector Addition
We can now define vector addition. The sum of two vectors, A and B, is a vector C, which is obtained by placing the initial point of B on the final point of A, and then drawing a line from the initial point of A to the final point of B , as illustrated in Panel 4. This is sometines referred to as the "Tip-to-Tail" method.

The operation of vector addition as described here can be written as C = A + B

3.9

Subtraction Vector subtraction is defined in the following way. The difference of two vectors, A B , is a vector C that is, C = A B

or C = A + (-B).Thus vector subtraction can be represented as a vector addition.

The graphical representation is shown in Panel 5. Inspection of the graphical representation shows that we place the initial point of the vector -B on the final point the vector A , and then draw a line from the initial point of A to the final point of -B to give the difference C.

Any quantity which has a magnitude but no direction associated with it is called a scalar". For example, speed, mass and temperature.

The product of a scalar, m say, times a vector A , is another vector, B, where B has the same direction as A but the magnitude is changed, that is, |B| = m|A|.

Many of the laws of ordinary algebra hold also for vector algebra. These laws are:

Commutative Law for Addition: A + B = B + A

Associative Law for Addition: A + (B + C) = (A + B) + C

The verification of the Associative law is shown in Panel 6.

If we add A and B we get a vector E. And similarly if B is added to C , we get F .

Now D = E + C = A + F. Replacing E with (A + B) and F with (B + C), we get (A +B) + C = A + (B + C) and we see that the law is verified.

Stop now and make sure that you follow the above proof.

Commutative Law for Multiplication: mA = Am

Associative Law for Multiplication: (m + n)A = mA + nA, where m and n are two different scalars.

Distributive Law: m(A + B) = mA + mB

These laws allow the manipulation of vector quantities in much the same way as ordinary algebraic equations.

3.10 Unit Vector


Vectors can be related to the basic coordinate systems which we use by the introduction of what we call "unit vectors."

A unit vector is one which has a magnitude of 1 and is often indicated by putting a hat (or circumflex) on top of the vector symbol, for

example

.The quantity

is read as "a hat" or "a unit".

Let us consider the two-dimensional (or x, y)Cartesian Coordinate System, as shown in Panel 7.

We can define a unit vector in the x-direction by by . Similarly in the y-direction we use

or it is sometimes denoted . Any two-dimensional

or sometimes

vector can now be represented by employing multiples of the unit vectors, and , as illustrated in Panel 8.

The vector A can be represented algebraically by A = Ax + Ay. Where Ax and Ay are vectors in the x and y directions. If Ax and Ay are the magnitudes of Ax and

Ay, then Ax respectively.

and Ay

are the vector components of A in the x and y directions

The breaking up of a vector into it's component parts is known as resolving a vector. Notice that the representation of A by it's components, Ax and Ay is

not unique. Depending on the orientation of the coordinate system with respect to the vector in question, it is possible to have more than one set of components.

The breaking up of a vector into it's components, makes the determination of the length of the vector quite simple and straight forward.

Since A = Ax

+ Ay

then using Pythagorus' Theorem

For example

. The resolution of a vector into it's components can be used in the addition and subtraction of vectors.

To illustrate this let us consider an example, what is the sum of the following three vectors?

Until now, we have discussed vectors in terms of a Cartesian, that is, an x-y coordinate system. Any of the vectors used in this frame of reference were directed along, or referred to, the coordinate axes. However there is another coordinate system which is very often encountered and that is the Polar Coordinate System.

It is possible to define fundamental unit vectors in the Polar Coordinate system in much the same way as for Cartesian coordinates. We require that the unit vectors be perpendicular to one another, and that one unit vector be in the direction of increasing r, and that the .other is in the direction of increasing

The multiplication of two vectors, is not uniquely defined, in the sense that there is a question as to whether the product will be a vector or not. For this reason there are two types of vector multiplication.

First, the scalar or dot product of two vectors, which results in a scalar.

And secondly, the vector or cross product of two vectors, which results in a vector.

In this tutorial we shall discuss only the scalar or dot product.

Note that the result of a dot product is a scalar, not a vector.

The rules for scalar products are given in the following

list,

And in particular we have itself is 0 and the cosine of 0 is 1.

, since the angle between a vector and

Alternatively, we have cosine of 90 is 0.

, since the angle between

and

is 90 and the

In general then, if AB = 0 and neither the magnitude of A nor B is 0, then A and B must be perpendicular.

3.11 Summary
If we are only interested in Abelian groups, we use + as the symbol for the group operation, 0 for the group identity, and -g for the inverse of g. This agrees with the additive notation in a ring. Indeed, the additive group of a ring is an Abelian group, and every Abelian group is the additive group of a ring. [To see this, take the group operation as addition, and construct the zero ring: all products are zero.]

3.12 Keywords
Group: A group is a structure with just one binary operation, satisfying four axioms. So groups are only half as complicated as rings.

G0) (Closure law) For any

, we have

(G1) (Associative law) For any

we have (g o h) o k = g o (h o k).

(G2) (Identity law) There is an element e EG with the property that goe = eog = g for all. (The element e is called the identity element of G.)

(G3) (Inverse law) For any element

there is an element

satisfying

e. (We denote this element h by ofg.)

and call it the inverse

If a group G also satisfies the condition

(G4) (Commutative law) For any g,h E G, we have goh = hog, then G is called a commutative group or (more often) an Abelian group.

3.13 Self Assessment Questions


1. Give the Definition of Group. 2. Explain Examples of Groups. 3. Discuss Properties of Groups. 4. Describe Uniqueness of Identity Element. 5. Explain Symmetric Groups. 6. Elaborate Subgroups. 7. Discuss Cyclic groups. 8. Define Vectors.

3.14 References
Elementary Engineering Mathematics by Dr. B.S. Grewal, Khanna Publications Higher Engineering Mathematics by B.S. Grewal, Khanna Publications Differential Calculus by Shanti Narayan, Publishers S. Chand & Co. Integral Calculus by Shanti Narayan, Publishers S. Chand & Co. Modern Abstract Algebra by Shanti Narayan, Publishers S. Chand & Co.

Unit 4 Limits and Continuity


Structure

4.0 4.1 4.2 4.3

Objectives Introduction Definition of Limits A Function with a jump 4.3.1 Another type of jump: the Heaviside function

4.4 4.5 4.6 4.7 4.8 4.9

Basic Properties of Limits Continuity Summary Keywords Questions References

4.0

Objectives
After studying this unit you will be able to : Define Limit Discuss various types of limits Define Continuity Discuss Continuity at a Point

4.1

Introduction
Let us understand that the concept of a limit is fundamental to Calculus. In fact, Calculus without limits is like Romeo without Juliet. It is at the heart of so many Calculus concepts like the derivative, the integral, etc. So what is a limit?

Maybe the best example to illustrate limits is through average and instantaneous speeds: Let us assume you are traveling from point A to point B while passing through point C. Then we know how to compute the average speed from A to B: it is simply the ratio between the distance from A to B and the time it takes to travel from A to B. Though we know how to compute the average speed this has no real physical meaning.

Indeed, let us suppose that a policeman is standing at point C checking for speeders going through C. Then the policeman does not care about the average speed. He only cares about the speed that you see on the speedometer, the one that the car actually has when crossing C. That one is real.

How do we compute this "instantaneous speed"? That's not easy at all! Naturally one way to do this is to compute the average speed from C to points close to C. In this case, the distance between these points and C is very small as well as the time taken to travel from them to C. Then we look at the ratio: Do these average speeds over small distances get close to a certain value? If so, that value should be called be the instantaneous speed at C. In fact, this is exactly how the policeman's radar computes the driver's speed!

4.2

Definition of Limits
Let us express this more mathematically. If s(t) is a function that determines the position of the moving object, and assume that at time t0, the moving object is at C. At , we are at a point close to C. Then the average speed between

these two points is

Then we study these numbers when

gets smaller and smaller. This is exactly

the idea behind the concept of limit. We will write

to indicate the instantaneous speed at C.

Before we state the formal definition of the limit, let us consider the function

What is

Clearly this function makes sense as long as the input is not equal to 0. In other words, we can take as an input any number close enough to 0, but not 0 itself.

It is clear by looking at the outputs that, when x gets close to 0,

is getting

close to 1. We say that

has a limit of 1 when x goes to 0 and write

You have to be very careful when you use calculators not to jump to conclusions too quickly. Quantities may be getting close to each other up to a certain point but then they may move further away from each other again. This happens frequently when dealing with chaotic systems, for example. Most of the calculators do computations up to nine digits or so. So two numbers with the same nine decimals are equal (according to the calculator). Be aware of the dangers from these shortcomings of calculating devices! But in the above

statement, we mean that

is getting as close to 1 as we wish. Of course, if

you want to get close up to 75 decimals then you will have to consider inputs x extremely close to 0. In other words, for a given error , then if x is close

enough to 0, we will have

is getting close to 1 up to

, or equivalently

How do we express: "x very close to 0"? Simply by saying that there exists such that then . Of course, as we said before, if is very small,

will usually have to be very small. In other words,

, which controls how

close we should be to 0, depends on how fast the function closer to 1, and on the size of

is getting

. Putting these ideas together, one can come up

with the following formal definition. Definition of limit. Let f(x) be a function defined around a point c, maybe not at c itself. We have

if for any

, there exists

such that

The number L is called the limit of f(x) when x goes to c.

Sometimes the function is not defined around the point c but only to the left or right of c. Then we have the concepts of left-limit and right-limits at c.

(i) L is the left-limit of f(x) at c iff for any

, there exists

such that

and write

(ii) L is the right-limit of f(x) at c iff for any

, there exists

such that

and write

Of course, if a function has a limit when x get closer to cfrom both sides then the left and right limits exists and are equal to the limit at the point, i.e. if exists then

4.3

A Function with a jump


Consider the function f(x) defined as having the value (2x2 - 2)/(x - 1) for with f(1) = 1. What is the limit of f(x) as x approaches 1? ,

Since the limit as

is based on values of f(x) for all x near a, but not equal


2

to a, only the formula (2x - 2)/(x - 1) matters! And since it is equal to 2x + 2 for all x values near 1, the value is near 21 + 2 = 4 there, and the limit is 4: limx 1 f(x) = 4, not 1.

Note well: The limit of f(x) as x goes to a does not always equal the value f(a), even when f(a) makes sense!

The graph of this function has a jump at x = 1, but the limit calculation ignores this, and treats the function as as if it were uninterrupted or continuous there.

4.3.1 Another type of jump: the Heaviside function EXAMPLE. In the physical description of sudden changes, like turning on a power switch, the Heaviside Function is often useful:

For t near 0 and positive, H(t) is 1, suggesting a limit of 1. But for t near 0 and negative, H(t) is 0, suggesting a limit of 0.

The limit cannot be both zero and one, so again this function has no limit ast^0, due to this jump from one value to another, which breaks the graph at this point. To start out with, let us note that the limit, when it exists, is unique. That is why we say "the limit", not "a limit". This property translates formally into:

Most of the examples studied before used the definition of the limit. But in general it is tedious to find the circumvent this. given the . The following properties help

4.4

Basic Properties of Limits


Theorem. Let f(x) and g(x) be two functions. Assume that

Then

(1)

(2)

, where

is an arbitrary number;

(3)

These properties are very helpful. For example, it is easy to check that

for any real number a. So Property (3) repeated implies

and Property (2) implies

These limits combined with Property (1) give

for any polynomial function

The next natural question then is to ask what happens to quotients of functions.

The following result answers this question:

Theorem. Let f(x) and g(x) be two functions. Assume that

Then

provided

This implies immediately the following:

where P(x) and Q(x) are two polynomial functions with

Example. Assume that

Find the limit

Answer. Note that we cannot apply the result about limits of quotients directly, since the limit of the denominator is zero. The following manipulations allow to circumvent this problem. We have

Using the above properties we get

and

Hence

which gives the limit

4.5

Continuity
We have seen that any polynomial function P(x) satisfies:

for all real numbers a. This property is known as continuity.

Definition. Let f(x) be a function defined on an interval around a. We say that f(x) is continuous at a iff

Otherwise, we say that f(x) is discontinuous at a.

Note that the continuity of f(x) at a means two things:

(i)

exists,

(ii) and this limit is f(a).

So to be discontinuous at a, means

(i)

does not exist,

(ii) or if

exists, then this limit is not equal to f(a).

Basic properties of limits imply the following:

Theorem. If f(x) and g(x) are continuous at a. Then (1) f(x) + g(x) is continuous at a;

(2)

is continuous at a, where

is an arbitrary number;

(3)

is continuous at a;

(4)

is continuous at a, provided

(5) If f(x) is positive, i.e.

, then

is continuous at a;

(6) If f(x) is continuous at a and g(x) is continuous at f(a), then their composition is continuous at a.

Remark. Many functions are not defined on open intervals. In this case, we can talk about one-sided continuity. Indeed, f(x) is said to be continuous from the left at a iff

and f(x) is said to be continuous from the right at a iff

Example. The function

is defined for

. So we can not talk

about left-continuity of f(x) at 0. But since we conclude that f(x) is rightcontinuous at 0.

This concept is also important for step-functions.

Example. Consider the function

The details are left to the reader to see

and

So we have

Since f(2) = 5, then f(x) is not continuous at 2.

Definition. For a function f(x) defined on a set S, we say that f(x) is continuous on S iff f(x) is continuous for all .

Example. We have seen that polynomial functions are continuous on the entire set of real numbers. The same result holds for the trigonometric functions and .

The following two exercises discuss a type of functions hard to visualize. But still one can study their continuity properties.

Example . Discuss the continuity of

Answer. Let us show that for any number a, the limit Indeed, assume otherwise that

does not exist.

Then from the definition of the limit implies that for any , such that

, there exists

Set

. Then exists

, such that

or equivalently

Since any open interval contains a rational and an irrational numbers, then we should have

Combining the two inequalities we get

which leads to an obvious contradiction. Thus the function is discontinuous at every point a.

Example . Let us modify the previous function: Discuss the continuity of

for

. (Two natural numbers p and q are coprime, if their greatest

common divisor equals 1.)

Answer. Let

. Let us show that

You may wonder how we guessed the value of this limit? This has to do with the fact that any real number may be approximated by irrational numbers: If the limit of f(x) exists, it has to be 0, since f(x)=0 for all irrational numbers in (0,1).

Let

. Consider the set

The crucial observation is that the set

is finite. For any

, the set of

natural numbers p such that

is finite. Set

The above findings imply that not in . In particular, we have

and if

satisfies, then q is

It is now easy to check that

for any

satisfying that.

The proof of our claim is complete.

Back to our original question. Since

for every irrational number every irrational point of (0,1).

, we conclude that f(x) is continuous at

Similarly, since

for every rational number every rational point of (0,1).

, we conclude that f(x)is discontinuous at

In the animation above, you see all points on the "graph" of f(x), whose ycoordinate exceeds 1/q (for increasing q). All other points on the "graph" of the function lie between the x-axis and the dashed line!

Definition (Continuity at one point). The function f is continuous at a if

If not, we say that f is discontinuous at a.

Note that this requires f to be defined at a.

4.6

Summary
Quantities may be getting close to each other up to a certain point but then they may move further away from each other again. This happens frequently when dealing with chaotic systems, for example. We see that there is no numerical value that f(x) gets close to, but there is a trend worth noting: the values of f(x) get larger and larger, with no upper bound. We have seen that many common functions f like polynomials have the nice property that the limit as x goes to a can be evaluated by simple evaluation of f(a). If the limit of f(x) exists, it has to be 0, since f(x)=0 for all irrational numbers in (0,1).

4.7

Keywords
Continuity: We have seen that any polynomial function P(x) satisfies:

for all real numbers a. This property is known as continuity.

4.8

Self Assessment Questions


1. Explain a Function with a jump.

2. Discuss the Properties of Limits. 3. Define Continuity. 4. What is Continuity at a point?

4.9

References
Elementary Engineering Mathematics by Dr. B.S. Grewal, Khanna Publications Higher Engineering Mathematics by B.S. Grewal, Khanna Publications Differential Calculus by Shanti Narayan, Publishers S. Chand & Co. Integral Calculus by Shanti Narayan, Publishers S. Chand & Co. Modern Abstract Algebra by Shanti Narayan, Publishers S. Chand & Co. Foundations of Differential Calculus - by Leonhard Euler, Euler, John D Blanton 215 pages A Treatise on the Differential Calculus: With ... - by Isaac Todhunter - 837 pages Differential and Integral Calculus - by Richard Courant

KARNATAKA STATE OPEN UNIVERSITY


MANASAGANGOTRI, MYSORE-570006

MCA 11
MATHEMATICS
BLOCK 2 DIFFERENTIAL CALCULUS

In collaboration with

VIRTUAL EDUCATION TRUST


Registered Office
UU -11, Pitampura, Delhi 110 034 Ph : 011-27348396 / 27348107 / 27345121 Fax : 011-27345042 E : vet@pctiltd.com, W : http://www.vet.pctiltd.com

First Edition 2009

Copyright by Virtual Education Trust All right reserved

No part of this publication may be reproduced in any form or by any means without the written permission from the Virtual Education Trust

All product names and company names mentioned

herein are the property of their respective owner.

Not for sale For personaluse of KSOU Informationtechnology&Management programme student only

Course Introduction

Differential calculus, a field in mathematics, is the study of how functions change when their inputs change. The primary object of study in differential calculus is the derivative. A closely related notion is the differential. The derivative of a function at a chosen input value describes the behavior of the function near that input value. For a real-valued function of a single real variable, the derivative at a point equals the slope of the tangent line to the graph of the function at that point. In general, the derivative of a function at a point determines the best linear approximation to the function at that point.

The process of finding a derivative is called differentiation. The fundamental theorem of calculus states that differentiation is the reverse process to integration.

Differentiation has applications to all quantitative disciplines. In physics, the derivative of the displacement of a moving body with respect to time is the velocity of the body, and the derivative of velocity with respect to time is acceleration. Newton's second law of motion states that the derivative of the momentum of a body equals the force applied to the body. The reaction rate of a chemical reaction is a derivative. In operations research, derivatives determine the most efficient ways to transport materials and design factories. By applying game theory, differentiation can provide best strategies for competing corporations.

Subject Introduction
In this block we will learn that Derivatives are frequently used to find the maxima and minima of a function. Equations involving derivatives are called differential equations and are fundamental in describing natural phenomena. Derivatives and their generalizations appear in many fields of mathematics, such as complex analysis, functional analysis, differential geometry, measure theory and abstract algebra. Unit 1 Summary, Keywords, Self assessment Questions, References. Physical Concept of the Derivative, The Geometrical,Concept of the Derivative, Using the Definition to Compute the Derivative, Derivatives and Rates of Change, Tangents, Velocities and Interpretation of the Derivative as the Slope of a Tangent Line, Summary, Keywords, Self assessment Questions, References.

Unit 2 Objectives, Introduction, Derivatives of Polynomial and Exponential Functions, Constant Multiples, Sums and Differences, Derivatives of Other Power Functions, Derivative of the Natural Exponential Function, The Derivatives of Trigonometric Functions, Derivatives Using the Limit Definition and Techniques of Differentiation,Summary, Keywords, Self assessment Questions, References.

Unit 3 Objectives, Introduction, Product Rule, Quotient Rule,The Chain Rule, Differentiation Using the Chain Rule, Inverse Functions in general and The Inverse Trigonometric Functions Summary, Keywords, Self

assessment Questions, References.

Unit 4 Objectives,

Introduction,

Differentiating

Implicitly

Defined

Functions,Parametric Representation, Differentiation in Parametric Form, Exponential functions, Properties of the Exponential Function and Hyperbolic Functions Differentiating Implicitly Defined Functions, First Order Linear Equations, Separable Equations, Bernoulli Equations, Homogeneous Equations, Exact and Nonexact Equations, Summary, Keywords, Self assessment Questions, References.

Mathematics Differential Calculus

Unit 1 The Concepts of Derivatives 5

Unit 2 Rules of Differentiation 25

Unit 3 Product Rule and Quotient Rule 42

Unit 4 Parametric Form and Exponential Functions 57

Unit 1 The Concepts of Derivatives


Structure

1.0 1.1 1.2 1.3 1.4 1.5

Objectives Introduction The Physical Concept of the Derivative The Geometrical Concept of the Derivative Using the Definition to Compute the Derivative Derivatives and Rates of Change 1.5.1 Tangents 1.5.2 Velocities

1.6 1.7 1.8 1.9 1.10

Interpretation of the Derivative as the Slope of a Tangent Line Summary Keywords Questions References

1.0

Objectives
After studying this unit you will be able to: Define Derivatives Discuss the Physical Concept of the Derivative Explain the Geometrical Concept of the Derivative Discuss to Compute the Derivative Explain Derivatives and Rates of Change Discuss Interpretation of the Derivative as the Slope of a Tangent Line

1.1

Introduction
Let us understand the concept of Derivative is at the core of Calculus and modern mathematics. The definition of the derivative can be approached in two different ways. One is geometrical (as a slope of a curve) and the other one is physical (as a rate of change). Historically there was (and maybe still is) a fight between mathematicians which of the two illustrates the concept of the derivative best and which one is more useful. We will not dwell on this and will introduce both concepts. Our emphasis will be on the use of the derivative as a tool.

1.2

The Physical Concept of the Derivative


This approach was used by Newton in the development of his Classical Mechanics. The main idea is the concept of velocity and speed. Indeed, assume

you are traveling from point A to point B, what is the average velocity during the trip? It is given by

If we now assume that A and B are very close to each other, we get close to what is called the instantaneous velocity. Of course, if A and B are close to each other, then the time it takes to travel from A to B will also be small. Indeed, assume that at time t=a, we are at A. If the time elapsed to get to B is we will be at B at time average velocity is . If , then

is the distance from A to B, then the

The instantaneous velocity (at A) will be found when

get smaller and smaller.

Here we naturally run into the concept of limit. Indeed, we have

If f(t) describes the position at time t, then case, we have

. In this

Example. Consider a parabolic motion given by the function f(t) = t2. The instantaneous velocity at t=a is given by

Since

we conclude that the instantaneous velocity at t=a is 2 a.

This concept of velocity may be extended to find the rate of change of any variable with respect to any other variable. For example, the volume of a gas depends on the temperature of the gas. So in this case, the variables are V (for

volume) as a function of T (the temperature). In general, if we have y = f(x), then the average rate of change of y with respect to x from x = a to where , is ,

As before, the instantaneous rate of change of y with respect to x at x = a, is

Notation. Now we get to the hardest part. Since we can not keep on writing "Instantaneous Velocity" while doing computations, we need to come up with a suitable notation for it. If we write dx for small, then we can use the notation

This is the notation introduced by Leibniz. (Wilhelm Gottfried Leibniz (1646-1716) and Isaac Newton (1642-1727) are considered the inventors of Calculus.)

1.3

The Geometrical Concept of the Derivative


Consider a function y = f(x) and its graph. Recall that the graph of a function is a set of points (that is (x,f(x)) for x's from the domain of the function f). We may draw the graph in a plane with a horizontal axis (usually called the x-axis) and a vertical axis (usually called the y-axis).

Fix a point on the graph, say (x0, f(x0)). If the graph as a geometric figure is "nice" (i.e. smooth) around this point, it is natural to ask whether one can find the equation of the straight line "touching" the graph at that point. Such a straight line is called the tangent line at the point in question. The concept of tangent may be viewed in a more general framework.

(Note that the tangent line may not exist. We will discuss this case later on.) One way to find the tangent line is to consider points (x,f(x)) on the graph, where x is very close to x0. Then draw the straight-line joining both points (see the picture below):

As you can see, when x get closer and closer to x0, the lines get closer and closer to the tangent line. Since all these lines pass through the point (x0,f(x0)), their equations will be determined by finding their slope: The slope of the line passing through the points (x0,f(x0)) and (x,f(x)) (where ) is given by

The tangent itself will have a slope m, which is very close to m(x) when x itself is very close to x0. This is the concept of limit once again!

In other words, we have

So the equation of the tangent line is

Notation. Writing "m" for the slope of the tangent line does not carry enough

information; we want to keep track of the function f(x) and the point x0 in our notation. The common notation used is

m = f'(x0).

In this case, the equation of the tangent line becomes

y - f(x0) = f'(x0) (x-x0)

where

One last remark: Sometimes it is more convenient to compute limits when the variable approaches 0. One way to do that is to make a translation along the xaxis. Indeed, if we set h=x-x0, we get

1.4

Using the Definition to Compute the Derivative


We have seen in the previous page how the derivative is defined: For a function f(x), its derivative at x=a is defined by

Let us give some examples.

Example . Let us start with the function f(x) = x2. We have

So

which means f '(a) = 2a.

What about the derivative of f(x) = xn. Similar calculations, using the binomial expansion for (x+y)n (Pascal's Triangle), yield

Example . Consider the function f(x)=1/x for

. We have

Consequently,

Have you noticed? The algebraic trick in both of the examples above has been to factor out "h" in the numerator, so that we can cancel it with the "h" in the denominator! This is what you try to do whenever you are asked to compute a derivative using the limit definition.

You may believe that every function has a derivative. Unfortunately that is not the case.

Example. Let us discuss the derivative of f(x) = |x| at 0. We have

But

which implies that f '(0) does not exist.

Remark. This example is interesting. Even though the derivative at the point does not exist, the right and the left limit of the ratio do exist. In fact, if we use the slope-interpretation of the derivative we see that this means that the graph has two lines close to it at the point under consideration. They could be seen as "halftangents". See Picture.

So let's push it a little bit more and ask whether a function always has a tangent or half-tangents at any point. That is not the case either.

Example . Let us consider the function = 0. We have

for

, with f(0)

Recall that the function

has no limit when x goes to 0. So the function

has no derivative and no half-derivatives as well at x=0.

What else can go wrong?

Example5. Consider the function

. Then we have

Since

then f '(0) does not exist. But observe that the graph as a geometric figure has a tangent -- albeit vertical:

In fact, the way the concept of the tangent line was introduced is based on the notion of slope. You already know that vertical lines do not have slopes. So we say that the derivative does not exist whenever the tangent line is vertical. Nevertheless keep in mind that when the limit giving the derivative is the function has a vertical tangent line at the point. then

It can be quite laborious (or impossible) to compute the derivative by hand as we have done so far. In the next pages we will show how techniques of differentiation help bypass the limit calculations and make our life much easier.

Example. Find the derivative of

Answer. Let us see how we can simplify the difference quotient

Rationalizing the numerator leads to

Consequently

Example. Discuss the differentiability of

Answer. May be the scariest thing about this function is the absolute value. So the best thing to do is to look for ways to remove it. Therefore we are led to find out when x2 - x is positive or negative. We get

Clearly the derivative exists at every point, except maybe at 0 and 1. Let us discuss these two points. Let us start with 0. We have

Since the function is defined differently from the left and the right of 0, then we have to consider the limits to the left and to the right at 0. We have

and

This implies that f'(0) does not exist. Similar computations will also give

and

which implies that f'(1) does not exist.

Example. We say that the graph of f(x) has a cusp at (a,f(a)), if f(x) is continuous at a and if the following two conditions hold:

1.

as

from one side (left or right);

2.

as

from the other side.

Determine whether f(x) = x4/3 and g(x) = x3/5 have a cusp at (0,0).

Answer. . For

, we have

So

f(x) does not have a cusp at 0. In fact, the graph has a horizontal tangent line at (0,0).

For the function g(x), we have

In this case, we have

So again (0,0) is not a cusp for g(x). But in this case, the graph has a vertical tangent at this point. Remember that a vertical line does not have a slope. So the derivative of g(x)at 0 does not exist.

Example. Show that if f '(a) exists, then we have

Answer. We know that

Equivalently, we also have

Putting these two equations together we will get

But

which gives the desired relation.

Note that this formula is for example used by calculators to approximate f'(a) especially when f(x) is known for values of x near a.

In the same spirit as the example above, one can prove the following formula which involves the second derivative:

1.5

Derivatives and Rates of Change


1.5.1 Tangents In Calculus we saw that the slope of the secant line on the curve y = f(x) between a point P(a, f(a)) and another point Q(x, f(x)), is

x-a

This slope function (f(x) - f(a))/(x - a) is undefined at x = a, but often it has a removable discontinuity there.

We also saw there that it makes sense to define the slope of the curve at P as the limit of this secant slope as x a:

In many cases, the line of this slope m through P touches the curve but does not cross it, so we call it the tangent line to y = f(x) at point P(a, f(a)), or the tangent at x = a.

It is often convenient to let h = x - a, the horizontal increment, so that x = a + h and the tangent slope is given by

The formulation in terms of step size h is even more useful when the algebra gets more complicated. ADDED EXAMPLE A Find the slopes of the tangent line to y = f(x) = x at the points (1,1), (4,2) and (9,3), by computing it at a general point P(a, a).

Hint: use the rationalizing factor

1.5.2 Velocities We Know that average velocity is given by a formula like that for secant slope.

For an object whose position at time t is f(t), the average velocity over a time interval of duration h from time a to time a + h is

The instantaneous velocity at time a is the limit of this as the length of the time interval h approaches zero:

The Derivative

The quantity

has been seen to be important to computing

slopes, velocities and other rates of change, so it deserve a name, and a shorthand, f'(a):

Definition. The derivative of function f at a number a is the quantity

if this limit exists.

An alternative form is

1.6

Interpretation of the Derivative as the Slope of a Tangent Line


The quantity now called f'(a) has been seen as the slope m of the tangent line to a curve. We can now write that

Definition. The tangent line to curve y = f(x) at point P(a, f(a)) is

y = l(x) = f(a) + f'(a)(x - a)

Interpretation of the Derivative as a Rate of Change

The derivative of position as a function of time is velocity, or the (time) rate of change of position.

Likewise the derivative of a function is the rate of change of the value of the function value with respect to change in the value of its argument.

For any quantity y related to another quantity x by y = f(x), changing the value of x from x1 to x2 causes a change in y from y1 = f(x1) to y2 = f(x2), so that the change by in x causes a

change of by y = y2 - y1 in y. The difference quotient defined by

gives the average rate of change of y respect to x over the interval [x1, x2]. As we adjust x2 to approach x1, so that x approaches 0, this approaches the instantaneous rate of change of y with respect to x,

1.7

Summary
The definition of the derivative can be approached in two different ways. One is geometrical (as a slope of a curve) and the other one is physical (as a rate of change). This concept of velocity may be extended to find the rate of change of any variable with respect to any other variable. The derivative of a function is the rate of change of the value of the function value with respect to change in the value of its argument.

1.8

Keywords
The derivative of function f at a number a is the quantity

if this limit exists.

1.9

Questions
1. Explain the term Derivative. 2. Give the Geometrical Concept of the Derivative. 3. Explain the Geometrical Concept of the Derivative. 4. Discuss to Compute the Derivative. 5. Explain Derivatives and Rates of Change 6. Discuss Interpretation of the Derivative as the Slope of a Tangent Line.

1.10

References
Elementary Engineering Mathematics by Dr. B.S. Grewal, Khanna Publications Higher Engineering Mathematics by B.S. Grewal, Khanna Publications Differential Calculus by Shanti Narayan, Publishers S. Chand & Co. Integral Calculus by Shanti Narayan, Publishers S. Chand & Co. Modern Abstract Algebra by Shanti Narayan, Publishers S. Chand & Co. Foundations of Differential Calculus - by Leonhard Euler, Euler, John D Blanton 215 pages A Treatise on the Differential Calculus: With ... - by Isaac Todhunter - 837 pages Differential and Integral Calculus - by Richard Courant

Unit 2 Rules of Differentiation


Structure

2.0 2.1 2.2 2.3 2.4 2.5 2.6 2.7 2.8 2.9 2.10 2.11 2.12

Objectives Introduction Derivatives of Polynomial and Exponential Functions Constant Multiples, Sums and Differences Derivatives of Other Power Functions Derivative of the Natural Exponential Function The Derivatives of Trigonometric Functions Derivatives Using the Limit Definition Techniques of Differentiation Summary Keywords Self Assessment Questions References

2.0

Objectives
After studying this unit you will be able to: Discuss the Derivatives of Polynomial and Exponential Functions Explain Constant Multiples, Sums and Differences Describe Derivative of the Natural Exponential Function Discuss Derivatives of Other Power Functions Explain the Derivatives of Trigonometric Functions Describe Derivatives Using the Limit Definition Define Techniques of Differentiation

2.1

Introduction
Let us understand that our collection of efficiently methods for computing derivatives starts with polynomials and exponential functions. Much as with limits, we do this by first dealing with a few simple functions, and then using rules for handling constant multiples, sums and differences.

2.2

Derivatives of Polynomial and Exponential Functions


However, derivatives of products and quotients do not follow the same pattern as with limits, so we leave them until the next section.

Since the slope of a straight line y = mx + c is the constant m, it is easy to check that the derivative of f(x) = mx + c is m, for any constants m and c. It is often convenient to use notation directly with formulas, without naming the functions, so to illustrate several notations:

Theorem (Derivatives of Linear Functions). The derivative of the linear function f(x) = mx + c is

The two most basic special cases are when the function is a constant c or just x:

Theorem (The Power Rule). For any positive integer n,

This includes f(x) = x1 = x and f(x) = x = 1, cases seen above. We have also almost seen this for f(x) = x1 in examples above with quadratics. Rather than do

that example, let us look at n = 4, which hints at how to do this calculation for any n.

EXAMPLE Calculate the derivative of f(x) = x4, and give the tangent line at point P(2,16). It is convenient in this case to use the first formula for the derivative f'(a):

The numerator vanishes for x = a, so it has a factor x - a, and in fact the factorization is x4 - a4 =

[Check by expanding!] This gives

, That is, (x4)' = 4x3, in agreement with the Power Rule above for n = 4. The tangent line at P(2,16) has slope m = f'(2) = 4 23 = 32. Thus the tangent line is y 16 = 32(x - 2), or y = 32x - 48.

Proof of the Power Rule (method 1)

The key step is the factorization

This can be checked by expanding the right hand side, distributing the left hand factor:

because all the terms in between pair off and cancel out.

2.3

Constant Multiples, Sums and Differences


As with limits, we can build up polynomials from these power functions using constant multiples, sums and differences. And the derivatives of these three basic combinations are as simple as with limits. Theorem (The Constant Multiple Rule). If a differentiable function f is multiplied by a constant c, its derivative is multiplied by the same constant:

Theorem (The Sum Rule). The sum of two differentiable functions f and g is differentiable, with the sums derivative the sum ofsummands derivatives:

Theorem (The Difference Rule). The difference of two differentiable functions f and g is differentiable, with its derivative the difference of their derivatives:

Warning: The rules seen so far are the only ones that are as simple and guessable as for limits!

Example Compute the derivative of y = x8 + 12x5 - 4x2 + 10x3 - 6x + 5.

The same approach works for differentiating any polynomial. By the way, this shows that all polynomials are differentiable.

2.4

Derivatives of Other Power Functions


Example show that x has derivative . That is,

fits the power rule, but for power 1/2, not a positive integer. In fact, the rule works for all real powers:

Theorem (The Power Rule, Generalized Version). For any real number a,

This is most easily shown later when we know how to differentiate exponential functions and compositions of functions.

2.5

Derivative of the Natural Exponential Function


We already defined the number e so that the slope of y = ex at point (0,1) is 1. That is

This choice makes the derivative of ex simple:

So the natural exponential function is equal to its own derivative:

Theorem (Derivatives of Exponential Functions).

Further, for any a > 0, it can be shown that

Thus all exponential functions have a rate of change proportional to their current value. This fits for example with the simple exponential model of a population whose growth rate is proportional to its current size because the rates of births and deaths are both proportional to current population.

The result for the derivative of ax can be seen graphically by writing that

so

The effect of changing from f(x) to g(x) = f(kx) is to compress the graph horizontally by a factor of k, increasing the slope at corresponding points by a factor k: in terms of derivatives, g'(x) = kf'(kx). So the graph of ax is a compression of the graph of ex by factor ln 2 and

Later we will see a more rigorous way to compute this derivative, using a derivative rule for compositions.

2.6

The Derivatives of Trigonometric Functions

Trigonometric functions are useful in our practical lives in diverse areas such as astronomy, physics, surveying, carpentry etc. How can we find the derivatives of the trigonometric functions?

Our starting point is the following limit:

Using the derivative language, this limit means that also be used to give a related one which is of equal importance:

. This limit may

To see why, it is enough to rewrite the expression involving the cosine as

But

, so we have

This limit equals

and thus

In fact, we may use these limits to find the derivative of

and

at

any point x=a. Indeed, using the addition formula for the sine function, we have

So

which implies

So we have proved that

exists and

Similarly, we obtain that

exists and that

Since

, and

are all quotients of the functions

and quotient rule:

, we can compute their derivatives with the help of the

It is quite interesting to see the close relationship between (and also between and ).

and

From the above results we get

These two results are very useful in solving some differential equations.

Example Let function, we can rewrite

. Using the double angle formula for the sine

So using the product rule, we get

which implies, using trigonometric identities,

2.7

Derivatives Using the Limit Definition


The following problems require the use of the limit definition of a derivative, which is given by

They range in difficulty from easy to somewhat challenging. If you are going to try these problems before looking at the solutions, you can avoid common mistakes by making proper use of functional notation and careful use of basic algebra. Keep in mind that the goal (in most cases) of these types of problems is to be

able to divide out the

term so that the indeterminant form

of the

expression can be circumvented and the limit can be calculated.

Example : Use the limit definition to compute the derivative, f'(x), for

Solution:

(Algebraically and arithmetically simplify the expression in the numerator.)

(The term

now divides out and the limit can be calculated.)

. Example : Use the limit definition to compute the derivative, f'(x), for

Solution:

(Algebraically and arithmetically simplify the expression in the numerator.)

(Factor

from the expression in the numerator.)

(The term

now divides out and the limit can be calculated.)

2.8

Techniques of Differentiation
Maybe the easiest and most useful formulas are the ones that say that the derivative is linear:

Combined with the formula (xn)' = n xn-1, we see that every polynomial function has a derivative at any point.

Example. For P(x) = 1-2x + 3x4 -5 x6, we have

The next two formulas are the most powerful ones. They deal with the derivative of a product and a quotient. They are commonly called the product rule and the quotient rule. We have

In particular, we have

So, we have

which means that the formula (xr)' = r xr-1 is also valid for negative exponents.

Before we discuss the derivative of trigonometric functions, let us stop here and reflect a little bit more on polynomial functions. Indeed, we saw that the derivative of a polynomial function is also a polynomial function. So we can take another derivative and generate a new function. This function is called the second derivative. We can keep doing this as long as we want to. The functions obtained are called higher derivatives. The common notations used for them are

Example Find the derivative of the function

Is there a nice way to rewrite this derivative?.

Answer. We use the quotient rule. We have

which gives

Another way to write the formula is

2.9

Summary
We saw that the derivative of a polynomial function is also a polynomial function. So we can take another derivative and generate a new function. This function is called the second derivative. We can keep doing this as long as we want to. The functions obtained are called higher derivatives.

2.10 Keywords
Techniques of Differentiation:

Derivatives of Linear Functions: The derivative of the linear function f(x) = mx

+ c is

2.11 Questions
1. Describe the Derivatives of Other Power Functions. 2. Explain the Derivatives of Trigonometric Functions. 3. Explain Constant Multiples, Sums and Differences. 4. Describe Derivative of the Natural Exponential Function. 5. Describe Derivatives Using the Limit Definition . 6. Define Techniques of Differentiation .

2.12 References
Elementary Engineering Mathematics by Dr. B.S. Grewal, Khanna Publications Higher Engineering Mathematics by B.S. Grewal, Khanna Publications Differential Calculus by Shanti Narayan, Publishers S. Chand & Co. Integral Calculus by Shanti Narayan, Publishers S. Chand & Co. Modern Abstract Algebra by Shanti Narayan, Publishers S. Chand & Co. Foundations of Differential Calculus - by Leonhard Euler, Euler, John D Blanton 215 pages A Treatise on the Differential Calculus: With ... - by Isaac Todhunter - 837 pages

Differential and Integral Calculus - by Richard Courant

Unit 3 Product Rule and Quotient Rule


Structure

3.0 3.1 3.2 3.3 3.4

Objectives Introduction Product Rule Quotient Rule The Chain Rule 3.4.1 Differentiation Using the Chain Rule

3.5

Inverse Functions in general 3.5.1 The Inverse Trigonometric Functions

3.0

Objectives
After studying this unit you will be able to: Discuss the Product Rule and Quotient Rule Describe Differentiation Using the Chain Rule Explain the Inverse Trigonometric Functions

3.1

Introduction
Let us understand that the Calculus has two simple formulas to find the derivatives of the sums and differences of functions. The derivative of a product of functions is not the same thing as taking the derivative of each term and then multiplying them together.

3.2

Product Rule
The Product Rule states: If f(x) = u(x) . v(x), where u and v are differentiable functions of x, then f ' (x) = u(x) . v'(x) + v(x) . u'(x)

The preceding formula says that the derivative of a product of two functions is the first term times the derivative of the second term plus the second term times the derivative of the first term.

Example: Use the Product Rule to find the derivative of f (x) = (x 2 + x)(2x +1)

Step 1 Identify u(x), u'(x), v(x), and v'(x) u(x) = x 2 +x ,

Step 2 Plug in the values identified in Step 1 into the formula

Therefore, using the Product rule, the derivative of the function (x 2 + x)(2x +1) is 6 x 2 + 6x + 1.

Example :

Use the Product Rule to find the derivative of f(x) = xsinx

Step 1 Identify u(x), u(x), v(x), v(x) The derivative of x is 1 and the derivative of sinx is cosx

Step 2 Plug in the values identified in Step 1 into the formula

Therefore, using the Product rule, the derivative of the function (xsinx) is (xcosx + sinx)

Example : Use the Product Rule to find the derivative of f(x) =

Step 2 Plug in the values identified in Step 1 into the formula

3.3

Quotient Rule.
The quotient rule is a method of finding the derivative of a function that is the quotient of two other functions for which derivatives exist.

The Quotient Rule states: functions of x, and v(x) ,

where u and v are differentiable

then f (x) =

The preceding formula says that the derivative of a quotient is the denominator times the derivative of the numerator minus the numerator times the derivative of the denominator, all divided by the square of the denominator.

Example: Use the quotient rule to find f (x)

Step 1

Step 2 Plug in the values identified in Step 1 into the formula

Therefore, using the Quotient rule, the derivative of the function

Example : Use the quotient rule to find the derivative of f(x) = Step 1

Step 2 Plug in the values identified in Step 1 into the formula

3.4

The Chain Rule

3.4.1 Differentiation Using the Chain Rule As a motivation for the chain rule, consider the function

f(x) = (1+x2)10.

Since f(x) is a polynomial function, we know from previous pages that f'(x) exists. Naturally one may ask for an explicit formula for it. One tedious way to do this is to develop (1+x2)10 using the Binomial Formula and then take the derivative. Of course, it is possible to do this, but it won't be much fun. But what if we have to deal with (1+x2)100! Then I hope you agree that the Binomial Formula is not the way to go anymore.

So what do we do? The answer is given by the Chain Rule.

Example. Let us find the derivative of through some trigonometric identities. Indeed, we have

. One way to do that is

So we will use the product formula to get

which implies

Using the trigonometric formula

, we get

Once this is done, you may ask about the derivative of

? The answer

can be found using similar trigonometric identities, but the calculations are not as easy as before. Again we will see how the Chain Rule formula will answer this question in an elegant way.

In both examples, the function f(x) may be viewed as:

where g(x) = 1+x2 and h(x) = x10 in the first example, and

and

g(x) = 2x in the second. We say that f(x) is the composition of the functions g(x) and h(x) and write

The derivative of the composition is given by the formula

Another way to write this formula is

where

and u = g(x). This second formulation (due to

Leibniz) is easier to remember and is the formulation used almost exclusively by physicists.

Example. Let us find the derivative of

We have

, where g(x) = 1+x2 and h(x) = x100. Then the Chain

rule implies that f'(x) exists, which we knew since it is a polynomial function, and

Example. Let us find the derivative of

We have

, where g(x) = 5x and

. Then the

Chain rule implies that f'(x) exists and

In fact, this is a particular case of the following formula

The following formulas come in handy in many areas of techniques of integration.

3.5

Inverse Functions in general


If y = 3x - 1

then x = (y + 1)/3.

The first formula defines a function f (x) = 3x - 1

The second formula gives the inverse (reverse) of this function, which we usually denote by the very unfortunate notation f-1(x).

All that we are doing is switching round the input and output of the machine-putting in at the `out' end and getting out at the `in' end. The inverse function undoes the effect of the original function:

(Note the notational problem. Does f-1 mean the inverse function of f or does it mean 1/f? I'm afraid that the answer has to be `it depends on the context'. You hope that it is obvious which interpretation the author has in mind. If it is not, then find another author! We really should have a better notation but once these things settle down it is very difficult to change them.)

There are problems. We cannot always define an inverse in a straight forward way.

Consider the function f (x) = x2. What can we make of f-1? If we put x = 1 into the f machine we get out 1. If we put -1 into the machine we also get out 1. So what do we expect to get out of the inverse machine if we put in 1? You see the problem. This machine is not uniquely reversible.

The problem that we face here is described by the technical term one-to-one. A function f is one-to-one if you never have f (x) = f (y) when x y.

The function f (x) = x2 is NOT one-to-one because, for example, f (1) = f (- 1).

Nevertheless, we can say that, in some sense, g(x) = Certainly, most people would say that g undoes the effect of f

is an inverse for f.

but this is not quite correct as it stands. It becomes correct if we put on the extra condition that x is not allowed to be negative and insist that the result of the square root is not negative.

3.5.1 The Inverse Trigonometric Functions The Inverse Trigonometric Functions are meant to be the `inverses' of the standard trigonometric functions. There are obviously going to be difficulties in trying to define them because none of the standard trig functions are one-to-one.

We have to start somewhere so let me start by giving the `obvious' (and obviously wrong) definitions of the inverse trig functions. I will then try to put them right. The inverse functions of sin, cos and tan are called arcsin, arccos and arctan. The notations sin-1(x), cos-1(x) and tan-1(x) are also common, though they tend to be confusing to beginners.

arcsin(x) is the angle whose sine is x arccos(x) is the angle whose cosine is x arctan(x) is the angle whose tan is x

if y = sin x then x = arcsin y if y = cos x then x = arccos y if y = tan x then x = arctan y

That's what we want to mean by the inverse trig functions, but the definitions do not make sense as written, simply because the trigonometric functions are not one-to-one.

For example, sin 0 = 0 and sin or what?

= 0 and so on. So what is arcsin 0? Is it 0 or

We get round this difficulty by carefully restricting the values that the inverse trigonometric functions can take.

r7cm

Figure 3.1: Graph of sin x.

Let me start with sin x. This function is not one-to-one, but it becomes one-to-one if we restrict attention to the range It also takes all its possible

values in this range. If we make the range any wider then the function ceases to be one-to-one. If we make the range any narrower then the function does not take all its possible values.

Definition

arcsin(x) is the angle between -

and

whose sine is

For the cosine we have to pick a different range because cosine is not one-to-

one on the range

Instead, we choose the range

This works.

Definition

arccos(x) is the angle between 0 and

whose cosine is

The graphs of the inverse trigonometric functions are given in Figs. 3 .2 and Fig. 33.

Figure 3.2: Graph of arcsin(x).

Figure 3.3: Graph of arccos(x).

These show the general pattern for the graphs of inverse functions. All we are doing is reversing the original x and y axes -- which is the same thing as flipping the graph over the line y = x. The resulting graph is a genuine graph (doesn't give more than one y value for any given x value) provided that the original function has an inverse.

For the tan we can use more or less the same range as we did for sine, except that we have to leave out the end points defined at these values (it blows up). /2 and /2 because tan is not

Definition arctan(x) is the angle in the range

whose tan is x. (x can have any value.)

Other Notations: As commented earlier we also use the notations sin -1(x), cos-(x) and tan-1(x) for the inverse trig functions. This is traditional but dangerous, because of the possible confusion with 1/sin(x) etc. I actually have a copy of a computer programming manual that gives a program for calculating arctan and uses that `formula' arctan(x) = cos(x)/sin(x)!

Your pocket calculator will almost certainly be able to calculate inverse trig functions for you. It is usually a matter of pressing some kind of INV button before pressing the required trig function button.

3.6

Summary
The Calculus has two simple formulas to find the derivatives of the sums and differences of functions. The derivative of a product of functions is not the same thing as taking the derivative of each term and then multiplying them together. The quotient rule is a method of finding the derivative of a function that is the quotient of two other functions for which derivatives exist.

3.7

Keywords

Product Rule: The Product Rule states: If f(x) = u(x) . v(x), where u and v are differentiable functions of x, then f ' (x) = u(x) . v'(x) + v(x) . u'(x)

Quotient Rule: The quotient rule is a method of finding the derivative of a function that is the quotient of two other functions for which derivatives

The Inverse Trigonometric Functions: The Inverse Trigonometric Functions are meant to be the `inverses' of the standard trigonometric functions.

3.8

Questions
1. Discuss the various rules of Differentiation. 2. Describe the Inverse Trigonometric Functions. 3. What is Chain Rule? 4. Explain Differentiation Using the Chain Rule. 5. Describe Inverse Functions in general.

3.9

References
Elementary Engineering Mathematics by Dr. B.S. Grewal, Khanna Publications Higher Engineering Mathematics by B.S. Grewal, Khanna Publications Differential Calculus by Shanti Narayan, Publishers S. Chand & Co. Integral Calculus by Shanti Narayan, Publishers S. Chand & Co.

Modern Abstract Algebra by Shanti Narayan, Publishers S. Chand & Co. Foundations of Differential Calculus - by Leonhard Euler, Euler, John D Blanton 215 pages A Treatise on the Differential Calculus: With ... - by Isaac Todhunter - 837 pages Differential and Integral Calculus - by Richard Courant

Unit 4 Parametric Form and Exponential Functions


Structure

4.0 4.1 4.2 4.3 4.4 4.5

Objectives Introduction Differentiating Implicitly Defined Functions Parametric Representation Differentiation in Parametric Form Exponential functions 4.5.1 Properties of the Exponential Function

4.6 4.7 4.8 4.9 4.10 4.11 4.12 4.13 4.14

Hyperbolic Functions First Order Linear Equations Separable Equations Bernoulli Equations Homogeneous Equations Exact and Nonexact Equations Summary Keywords Self Assessment Questions

4.15

References

4.0

Objectives
After studying this unit you will be able:

Define Parametric Representation Describe Differentiating Implicitly Defined Functions Explain Exponential functions Discuss Hyperbolic Functions Define First Order Linear Equations Describe Separable Equations Explain Bernoulli Equations Exact and Nonexact Equations

4.1

Introduction
Let us understand that the variable y is a function of the variable x. Up to now the definition of y in terms of x has been given in the form

y = expression in $x$

This is called an Explicit Formula for y in terms of x. You get the value of y by plugging the value of x into the RHS.

It is also possible, with a bit of care, to define y in terms of x by a equation of more general form involving x and y. For example, the equation of a straight line:

ax + by + c = 0 or the equation of a circle:

x2 + y2 = r2

or, more elaborately,

x2y2 + y4 = sin(xy)

Such an equation is called an Implicit Formula for y in terms of x.

In some cases it is easy to turn an Implicit Equation into an Explicit Equation, simply by `solving for y'. For example, if b then the equation can be changed to 0 in the equation of the straight line,

y=-

x-

which is now explicit.

As another example, the Implicit Equation 3y + x2 = 2x2y can be rearranged to give the Explicit Equation

y=

Things are not always so simple. There are two problems that we face. The first and most obvious is that we may not know how to solve the equation for y -- see the third example at the start, you cannot do it and nor can I.

The second problem is more theoretical. An implicit equation on its own may not define y as a function of x in a straightforward way. Consider the example of the circle equation x2 + y2 = r2. If we `solve for y' we actually get two explicit equations:

y=

and

y=-

Since, within the range - r < x < r there are always two y-values for each x-value we cannot hope to express y as a single function of x.

So when we talk of the function defined by an Implicit Equation we should always be aware that there may be more than one and that we may need extra information to decide on which to choose.

4.2

Differentiating Implicitly Defined Functions


Suppose that y is a function of x that satisfies the Implicit Equation

x2 + y2 = sin(x + y)

Can we work out the derivative dy/dx?

This would be no problem if we could rearrange our equation into an Explicit equation y =..., but in this case we cannot. Even if we cannot convert the equation into explicit form it is still quite easy to work out the derivative by using the chain rule. Just differentiate both sides of the equation, remembering that y is a function of x and that we will have to use the Chain Rule on it.

One or two examples will show you how easy this is.

Example

x2 + y2 = 1

Differentiating both sides wrt x we get

2x +

(y2) = 0

By the chain rule

(y2) =

(y2).

So the equation becomes

2x + 2yy' = 0

or

=-

Note the almost inevitable snag: the expression for the derivative contains both x and y.

Example

Find the equation of the tangent line to the curve x2 + y3 = 2 at the

point (1, 1) on the curve. To find the equation of the tangent we need to find the slope of the curve at (1, 1). This means working out the derivative. We differentiate both sides of the equation with respect to x. The derivative of x2 is 2x and, by the chain rule, the derivative of y3 is 3y2y'. So we get

2x + 3y2y' = 0

hence

y' = -

So the slope of the curve at the required point is y' = - 2/3. So the equation of the tangent is y - 1 = (x - 1).

Example

x3 + sin(xy) = xy2

Differentiating both sides wrt x we get

3x2 +

sin(xy) = 1.y2 + x

y2

This still leaves us with some work to do:

y2 = 2yy'

as before.

The sin term involves a double application of the chain rule. We have, let's say, z = sin u where u = xy. So

= cos(u)

(xy)

Now

(xy) = 1.y + x.

y = y + xy'

Finally we get

3x2 + (y + xy')cos(xy) = y2 + 2xyy'

We can now rearrange this to get all the y' terms onto the LHS:

(x cos(xy) - 2xy)y' = y2 - 3x2 - y cos(xy)

So

--a mess, but a successful mess.

Example Find the tangent at (0, 0) to the curve (x2 + y2)2 = x2 - y2.

Differentiate implicitly to get

2(x2 + y2)(2x + 2yy') = 2x - 2yy'ory' =

Now put x = 0 and y = 0 -- and get problems: 0/0. It doesn't work.

What's happening? We are interested in the point (0, 0). Suppose that we are very close to it, so that x and y are very small. Then x2 and y2 are extremely small and (x2 + y2)2 is absolutely tiny. So, rather approximately, we can say that, near (0, 0), the equation is x2 - y2 0 or y2 x2 or y x. This gives two straight

lines through the origin at right angles--which explains why we had difficulty in finding the slope at that point. In fact the curve, a Lemniscate, is shown in Fig.4.1

Figure 4.1

Example Here is another kind of example of the use of implicit differentiation.

We are told that the function y(x) satisfies the equation values of x. We are also told that y(0) = 1.

+ xy2 = x + 1 for all

What, in that case, are y'(0), y''(0) and y'''(0)?

Put x = 0 and y = 1 into the equation and get y'(0) + 0 = 0 + 1. So y'(0) = 1. Differentiate the equation wrt x and get

y'' + y2 + 2xyy' = 1

Put x = 0 in this, together with y = 1 and y' = 1 and get y''(0) = 0.

Differentiate again wrt x and get

y''' + 4yy' + 2xy'2 + 2xyy'' = 0

Put x = 0 in this together with y = 1, y' = 1, y'' = 0 and get y'''(0) = - 4.

4.3

Parametric Representation

Suppose we have a point moving around in the (x, y) plane. At each time t the particle will be at some point whose coordinates we can write as (x(t), y(t)). i.e. the x and y coordinates of the point are given as functions of the parameter t.

Many curves can be most conveniently expressed in the form

x = x(t)

y = y(t)

where t is some parameter. t need not be time, though it often helps to think about it in that way. This is called a parametric representation of the curve.

It may in some cases be possible to eliminate t between the two equations and get an Implicit Equation just involving x and y. Even when possible, this is not always desirable.

Example The Circle Any point on the circle x2 + y2 = r2 can be written in the form

x( y(

) = r cos ) = r sin

-- just using polar coordinates.

This is now a parametric representation of this circle. As the point (x( ), y( )) goes once round the circle.

goes from 0 to 2

Similarly, the circle (x - a)2 + (y - b)2 = r2 can be represented parametrically as

x( y(

) = a + r cos ) = b + r sin

There are always lots of different ways of parameterizing a given curve, some more helpful than others. As a trivial example, x(t) = sin(t), gives a circle, as does x(t) = cos(t2), y(t) = sin(t2). y(t) = cos(t) still

Notice that a parametric representation can represent a whole curve in situations where is is not possible to do this by a formula of the form y = f (x), because there may be more than one value of y corresponding to a given value of x. This representation can even handle cases where a curve crosses over itself, as in the case

x(t) = cos(t),

y(t) = sin(2t)

which produces a shape like the

sign.

4.4

Differentiation in Parametric Form


Suppose that we are given a curve in the parametric form

x = x(t)

y = y(t)

where t is a parameter. Can we find the value of dy/dx (in other words, the slope of the curve) without actually having to eliminate t between the equations? The answer is yes and the process is simple and based on the following simple consequence of the chain rule:

hence,

where the dot denotes differentiation wrt t.

Note something that might be a problem in some cases: the answer is given in terms of the parameter t rather than in terms of x and y.

Example Consider the circle in the parametric form

x = r cos t y = r sin t

Then

= - r sin t And

= r cos t

so

= - cot t

Example The parabola y2 = 4ax can be parametrised as

x(t) = at2

y(t) = 2at

where t takes all values.

Why is this true? Firstly, y2 = 4a2t2 and 4ax = 4a2t2 so the point (x(t), y(t)) certainly satisfies the equation and lies on the parabola. That's half the story.

The second question is: do we get the whole parabola? i.e. is there a value of t corresponding to each point on the curve? This question is easy to answer in this case. Let (x, y) be a point on the parabola, so y2 = 4ax. Let t = y/2a. Then y = 2at and x = y2/4a = at2.

Final slight worry: do different values of t give us the same point on the parabola? No they do not -- as the above argument actually shows. Put another way: if (at2, 2at) = (as2, 2as) then we must have s = t, as can be seen by comparing the ycoordinates.

Having settled all that, let's differentiate:

= 2at

= 2a

so

To expand on the points raised in the previous example, consider the curve given parametrically by

x(t) = at4 y(t) = 2at2

where t takes all values.

What curve is this? Well, it is easy to see that x(t) and y(t) satisfy y2 = 4ax. So we seem to have the parabola again. No we don't. We get half the parabola. In this parametric representation, assuming that a > 0, the y value can never be

negative (2at2). Furthermore, we get half the parabola twice over: (at4, 2at2) = (as4, 2as2) if t = s.

If we think of (x(t), y(t)) as the path of a particle parametrised by time then the particle comes in from infinity along the top half of the parabola, gets to the origin, stops and then reverses back the way it came.

4.5

Exponential functions
Definition

Consider the following equation for a function y(x):

=y

y(0) = 1

This is just a simplified version of the growth law that we derived in the previous section (the growth rate is unity).

It can be proved that this equation has a solution that is defined and differentiable for all values of x. Please accept this.

Proposition 4.1 If y(x) is a solution to (4.1) then y(x)y(- x) = 1 for all values of x. Proof. This is easy to prove once you get the idea, and the idea is going to be used frequently in this section.

Consider the function A(x) = y(x)y(- x) where y(x) is a solution to (4.1).

Differentiating both sides of this equation we get, using (4.1):

A'(x) = y'(x)y(- x) - y(x)y'(- x) = y(x)y(- x) - y(x)y(- x) =0

by product and chain rules by (4.1)

So the derivative of A(x) is always zero. So A(x) is a constant. But when x = 0 we have A(0) = y(0)y(0) = 1. So A(x) = 1 for all values of x. That proves the proposition.

Proposition 4.2 If y(x) is a solution to (4.1) then y(x) is never zero.

Proof. This is obvious: if y(x) = 0 then we can hardly have y(x)y(- x) = 1.

Now we come to a very important result. Up until now I have talked loosely about `the' solution to equation (4.1). Could it actually have more than one solution (like quadratic equations can have more than one solution)? The common-sense of our population model strongly suggests that it cannot. If I know how large the population is at the start and also know the law for its growth then that really ought to fix the population thereafter. But common sense is not mathematical proof. So we need a theorem.

Proposition 4.3 The equation(4.1) has precisely one solution, defined for all x.

Proof. Thanks to the results that we have already proved this theorem is quite easy to prove. If you are trying to prove that at most one of a certain kind of thing exists then a standard approach is to use `proof by contradiction'. You assume the opposite--that there is more than one--and then try to show that this leads to a logical contradiction. If it does then you must have been right in thinking that there was at most one. In our case we already know that there is at least one solution, so we will have proved that there is exactly one.

So suppose that y1(x) and y2(x) are two different solutions to (4.1).

Consider the function

z(x) =

There is no danger of this function being undefined for any value of x because we have just proved that the denominator can never be zero.

Differentiate z(x) and get

z'(x) =

= =0

by (4.1)

So the derivative of z(x) is always zero, so z(x) is constant. When x = 0 we have z(0) = 1/1 = 1. So z(x) = 1 for all values of x and therefore

y1(x) = y2(x)

for all values of x. This contradicts our original assumption that y1 and y2 were different. So (4.1) can only have one solution.

Now we are getting places. Equation (4.1)definitely defines a single function of x.

The single solution to equation (4.1) is called the Exponential Function and is written as exp(x) or ex.

The second notation makes the exponential function look like a power. It is, but we have not proved that yet. So think of it as a peculiar notation for the time being.

The number exp(1) is denoted by e and has the value

There is almost certainly a button on your pocket calculator for working out exp(x). It will be called Exp or ex. Try using it to check some of the claims that have been made.

We have already proved the following facts about the exponential function:

i.e. it has the remarkable property that it does not change when you differentiate it.

The first one is just saying that y(x) = exp(x) satisfies the equation in (4.1), which it does by definition. The second one is saying that exp(x) satisfies the condition y(0) = 1 in (4.1), which is also true by definition. The third and fourth are consequences of our theorems.

4.5.1 Properties of the Exponential Function Now we push ever onwards. The next theorem is the really juicy one. It shows that the exponential function has a remarkably nice property that you would hardly guess at on the basis of equation (1). It goes a long way towards showing that exp(x) is `really' a power, as I said above.

Proposition4.4 For all values of x and y

exp(x + y) = exp(x)exp(y)

or

ex + y = exey

Proof. Same trick as usual. Pick any value of y. Now let

A(x) =

Differentiate this with respect to x (we are thinking of y as a constant):

A'(x) = =0 using chain rule and (4.1)

So A(x) has a constant value as x varies. Put x = 0 and get A(0) = exp y. So, for all values of x and y, we have

= exp y

and hence

exp(x + y) = exp(x)exp(y)

This is the crucial `law of indices' property of the exponential function.

Proposition 4.5 exp(x) is positive for all values of x. Proof. Using Prop. 4.4 we have exp(x) = exp(x/2)exp(x/2) = exp(x/2)2. So exp(x) cannot be negative and we have already shown that it cannot be zero either. So it is always positive.

Proposition 4.6

The exponential function exp(x) is always increasing and has

the following limiting behaviour:

Proof. We know that exp(x) is always positive, so equation (4.1) tells us that the derivative of exp(x) is always positive. So exp(x) is always increasing. We can prove that it tends to as x if we can prove that exp(x) takes arbitrarily

large values. This is easy, thanks to Prop 4.3. We know that exp(0) = 1, so exp(2) > 1. Now exp(4) = exp(2)exp(2) and exp(8) = exp(4)exp(4) and so on up. In general,

Since exp(2) > 1 its powers get bigger and bigger without limit. That's what we wanted.

On the other side: exp(- x) = 1/exp(x) so exp(- x) proves the second point.

0 as x

and that

That's the end of my basic work on the exponential function.

4.6

Hyperbolic Functions
There are some simple combinations of exponential functions that have been given special names. There is really nothing much to them at this level, but they should be known. The functions sinh x, cosh x and tanh x are defined as follows

cosh x =

(ex + e-x)

sinh x =

(ex - e-x)

tanh x =

They are pronounced as `cosh', `shine' and `tansh' (or `than') and are called the Hyperbolic Trigonometric Functions. You may have buttons for them on your calculator.

Their properties can be deduced easily from those of ex. You should check that

cosh(- x) = cosh(x)

sinh(- x) = - sinh(x)

tanh(- x) = - tanh(x)

cosh(0) = 1

sinh(0) = 0

as

cosh x = sinh x

sinh x = cosh x

cosh2x - sinh2x = 1

There are also formulas corresponding to all the usual trigonometric formulas, e.g. for sinh(x + y) and sinh 2x. They are the same as the trig formulas except for some of the signs.

Let us understand what the differential equation is.

A differential equation is an equation involving an unknown function and its derivatives. 2. The order of the differential equation is the order of the highest derivative of the unknown function involved in the equation. 3. A linear differential equation of order n is a differential equation written in the following form:

where

is not the zero function. Note that some may use the notation for the derivatives.

4.7

First Order Linear Equations

A linear equation obliges the unknown function y to have some restrictions. Indeed, the only operations which are accepted for the variable y are: (i) Differentiating y; (ii) Multiplying y and its derivatives by a function of the variable x (iii) Adding what you obtained in (ii) and let it be equal to a function of x. 4. Existence: Does a differential equation have a solution? 5. Uniqueness: Does a differential equation have more than one solution? If yes, how can we find a solution which satisfies particular conditions?

6. A problem in which we are looking for the unknown function of a differential equation where the values of the unknown function and its derivatives at some point are known is called an initial value problem (in short IVP). 7. If no initial conditions are given, we call the description of all solutions to the differential equation the general solution

A first order linear differential equation has the following form:

The general solution is given by

where

called the integrating factor. If an initial condition is given, use it to find the constant C. Here are some practical steps to follow: 1. If the differential equation is given as

rewrite it in the form

, where

2. Find the integrating factor

3. Evaluate the integral 4. Write down the general solution

. 5. If you are given an IVP, use the initial condition to find the constant C.

Example: Find the particular solution of:

Solution: Let us use the steps: Step 1: There is no need for rewriting the differential equation. We have

Step 2: Integrating factor

. Step 3: We have

. Step 4: The general solution is given by

. Step 5: In order to find the particular solution to the given IVP, we use the initial condition to find C. Indeed, we have . Therefore the solution is

4.8

Separable Equations

The differential equation of the form h(x) g(y); that is,

is called separable, if f(x,y) =

In order to solve it, perform the following steps: (1) Solve the equation g(y) = 0, which gives the constant solutions of (S); (2) Rewrite the equation (S) as

, and, then, integrate

to obtain

(3) Write down all the solutions; the constant ones obtained from (1) and the ones given in (2); (4) If you are given an IVP, use the initial condition to find the particular solution. Note that it may happen that the particular solution is one of the constant solutions given in (1). This is why Step 3 is important.

Example: Find the particular solution of

Solution: Perform the following steps:

(1)

In order to find the constant solutions, solve and y=-1.

. We obtain y = 1

(2)

Rewrite the equation as

. Using the techniques of integration of rational functions, we get

, which implies

(3)

The solutions to the given differential equation are

(4)

Since the constant solutions do not satisfy the initial condition, we are left to find the particular solution among the ones found in (2), that is we need to find the constant C. If we plug in the condition y=2 when x=1, we get

Note that this solution is given in an implicit form. You may be asked to rewrite it in an explicit one. For example, in this case, we have

Example: Find all solutions to

. Solution: First, we look for the constant solutions, that is, we look for the roots of

This equation does not have real roots. Therefore, we do not have constant solutions. The next step will be to look for the non-constant solutions. We proceed by separating the two variables to get

. Then we integrate

Since

= we get

=1-

Therefore, we have

It is not easy to obtain y as a function of t, meaning finding y in an explicit form. Finally, because there are no constant solutions, all the solutions are given by the implicit equation

4.9

Bernoulli Equations
A differential equation of Bernoulli type is written as

This type of equation is solved via a substitution. Indeed, let easy calculations give

. Then

which implies

This is a linear equation satisfied by the new variable v. Once it is solved, you will obtain the function . Note that if n > 1, then we have to add the

solution y=0 to the solutions found via the technique described above. Let us summarize the steps to follow: (1) Recognize that the differential equation is a Bernoulli equation. Then find the parameter n from the equation;

(2) Write out the substitution

(3) Through easy differentiation, find the new equation satisfied by the new variable You may want to remember the form of the new equation: v.

(4) Solve the new linear equation to find v;

(5) Go back to the old function y through the substitution (6) If n > 1, add the solution y=0 to the ones you obtained in (4).

(7) If you have an IVP, use the initial condition to find the particular solution. Example: Find all the solutions for

Solution: Perform the following steps: (1) We have a Bernoulli equation with n=3;

(2) Consider the new function (3) The new equation satisfied by v is

; (4) This is a linear equation:

4.1 the integrating factor is

4.2 we have 4.3 the general solution is given by

5 Back to the function y: we have

, which gives

6 All solutions are of the form

4.10 Homogeneous Equations


The differential equation

is homogeneous if the function f(x,y) is homogeneous, that is-

Check that the functions

. are homogeneous. In order to solve this type of equation we make use of a substitution (as we did in case of Bernoulli equations). Indeed, consider the substitution homogeneous, then we have . If f(x,y) is

Since y' = xz' + z, the equation (H) becomes

which is a separable equation. Once solved, go back to the old variable y via the equation y = x z. Let us summarize the steps to follow: (1) Recognize that your equation is an homogeneous equation; that is, you need to check that f(tx,ty)= f(x,y), meaning that f(tx,ty) is independent of the variable t; (2) (3) Write out the substitution z=y/x; Through easy differentiation, find the new equation satisfied by the new function You may want to remember the form of the new equation: z.

(4) (5) (6)

Solve the new equation (which is always separable) to find z; Go back to the old function y through the substitution y = x z; If you have an IVP, use the initial condition to find the particular solution.

Since you have to solve a separable equation, you must be particularly careful about the constant solutions. Example: Find all the solutions of

Solution: Follow these steps:

(1) It is easy to check that

is homogeneous;

(2) Consider (3) We have

, which can be rewritten as

This is a separable equation. If you don't get a separable equation at this point, then your equation is not homogeneous, or something went wrong along the way. (4) All solutions are given implicitly by

(5) Back to the function y, we get

Note that the implicit equation can be rewritten as

4.11 Exact and Nonexact Equations


All the techniques we have reviewed so far were not of a general nature since in each case the equations themselves were of a special form. So, we may ask, what to do for the general equation

Let us first rewrite the equation into

This equation will be called exact if

, and nonexact otherwise. The condition of exactness insures the existence of a function F(x,y) such that

When the equation (E) is exact, we solve it using the following steps: (1) Check that the equation is indeed exact; (2) Write down the system

(3) Integrate either the first equation with respect of the variable x or the second with respect of the variable y. The choice of the equation to be integrated will depend on how easy the calculations are. Let us assume that the first equation was chosen, then we get

The function

should be there, since in our integration, we assumed that the

variable y is constant.

(4) Use the second equation of the system to find the derivative of we have

. Indeed,

, which implies

Note that is a function of y only. Therefore, in the expression giving variable, x, should disappear. Otherwise something went wrong!

the

(5) Integrate to find

(6) Write down the function F(x,y); (7) All the solutions are given by the implicit equation

(8) If you are given an IVP, plug in the initial condition to find the constant C. You may ask, what do we do if the equation is not exact? In this case, one can try to find an integrating factor which makes the given differential equation exact.

4.12 Summary
An implicit equation on its own may not define y as a function of x in a straightforward way. Even if we cannot convert the equation into explicit form it is still quite easy to work out the derivative by using the chain rule. Just differentiate both sides of the equation, remembering that y is a function of x and that we will have to use the Chain Rule on it.

4.13

Keywords Differential Equation: A differential equation is an equation involving an unknown function and its derivatives.

4.14 Self Assessment Questions


1. Explain Differentiating Implicitly Defined Functions. 2. What do you mean by Parametric Representation?

3. Explain Exponential functions. 4. Discuss Hyperbolic Functions 5. Define First Order Linear Equations. 6. Describe Separable Equations. 7. Explain Bernoulli Equations.

4.15

References
Elementary Engineering Mathematics by Dr. B.S. Grewal, Khanna Publications Higher Engineering Mathematics by B.S. Grewal, Khanna Publications Differential Calculus by Shanti Narayan, Publishers S. Chand & Co. Integral Calculus by Shanti Narayan, Publishers S. Chand & Co. Modern Abstract Algebra by Shanti Narayan, Publishers S. Chand & Co.

KARNATAKA STATE OPEN UNIVERSITY

MANASAGANGOTRI, MYSORE-570006

MCA 11
MATHEMATICS
BLOCK 3 INTEGRAL CALCULUS

In collaboration with

VIRTUAL EDUCATION TRUST


Registered Office
UU -11, Pitampura, Delhi 110 034 Ph : 011-27348396 / 27348107 / 27345121 Fax : 011-27345042 E : vet@pctiltd.com, W : http://www.vet.pctiltd.com

First Edition 2009

Copyright by Virtual Education Trust All right reserved

No part of this publication may be reproduced in any form or by any means without the written permission from the Virtual Education Trust

All product names and company names mentioned

herein are the property of their respective owner.

Not for sale For personaluse of KSOU Informationtechnology&Management programme student only

Course Introduction

Engineering physics is a branch of applied science that emphasizes both engineering and physics. The engineering physics curriculum is designed to fulfill the educational requirements for professional work in various fields of applied science which are based upon a thorough knowledge of physics and foundation of basic scientific principles, as well as the theoretical knowledge and skills required for specific engineering applications.

The study of Engineering Physics emphasizes the application of basic scientific principles to the design of equipment, which includes electronic and electromechanical systems, for use in measurements, communications, and data acquisition. The course is recommended for students interested in newly developing areas of physics, high technology, instrumentation and communications.

Subject Introduction
This Block explains concepts of Integration, Integration of Rational functions, Substitution, Trigonometric substitutions and Convergence and Divergence of Improper Integrals,

Unit 1 Objective, Introduction, Integration by Parts, Integration of Rational functions, Substitution, Trigonometric substitutions, Rational Expressions of Trigonometric Functions, Summary, Keywords, Self Assessment Questions, References.

Unit 2 Objective, Introduction, Product of Sines and Cosines, Trigonometric Powers, Integration of Nonelementary Functions, Local behaviour of functions, Taylor Polynomials, Indeterminate Forms, Indeterminate

Quotient Forms, Indeterminate Quotient Form Self Assessment Questions, References.

, Summary, Keywords,

Unit 3 Objective, Introduction, Polynomial Long Division, Summary, Keywords, Self Assessment Questions, References.

Unit 4 Objective, Introduction, Convergence and Divergence of Improper Integrals, Summary, Keywords, Self Assessment Questions, References..

Mathematics Integral Calculus

Unit 1 Integration and Its Techniques 4

Unit 2 Integration of Trigonometric Functions 22

Unit 3 The Area Problem and the Definite Integral 40

Unit 4 Improper Integrals 55

Unit 1 Integration and Its Techniques


Structure

1.0 1.1

Objective Introduction 1.1.1 Properties of Integral 1.1.2 Integration by Parts

1.2 1.3 1.4 1.5 1.6 1.7 1.8 1.9

Integration of Rational functions Substitution Trigonometric substitutions Rational Expressions of Trigonometric Functions Summary Keywords Self Assessment Questions References

1.0

Objective
After studying this unit you will be able to: Define Techniques of Integration Discuss Integration of Rational functions Explain Trigonometric substitutions Discuss Substitution Explain Rational Expressions of Trigonometric Functions

1.1

Introduction

Given

the process of finding y is called Integration and the resulting

function is called Integral. If g (x) is represent the process.

is the notation used to

In the above notation (x) is called Integrand and further

But[g(x)

c]=g'(x)

when

is

constant

Thus integral of a function is not unique and two integrals always differ by a constant.

1.1.1

Properties of Integral

Standard Integrals

1.1.2 Integration by Parts Let us understand that one of very common mistake students usually do is

To convince yourself that it is a wrong formula, take f(x) = x and g(x)=1. Therefore, one may wonder what to do in this case. A partial answer is given by what is called Integration by Parts. In order to understand this technique, recall the formula

which implies

Therefore if one of the two integrals

and

is easy

to evaluate, we can use it to get the other one. This is the main idea behind Integration by Parts. Let us give the practical steps how to perform this technique: 1 Write the given integral

where you identify the two functions f(x) and g(x). Note that if you are given only one function, then set the second one to be the constant function g(x)=1.

Introduce the intermediary functions u(x) and v(x) as:

Then you need to make one derivative (of f(x)) and one integration (of g(x)) to get

Note that at this step, you have the choice whether to differentiate f(x) or g(x). We will discuss this in little more details later.

3 Use the formula

4 Take care of the new integral

The first problem one faces when dealing with this technique is the choice that we encountered in Step 2. There is no general rule to follow. It is truly a matter of experience. But we do suggest not to waste time thinking about the best choice, just go for any choice and do the calculations. In order to appreciate whether your choice was the best one, go to Step 3: if the new integral (you will be handling) is easier than the initial one, then your choice was a good one, otherwise go back to Step 2 and make the switch. It is after many integrals that you will start to have a feeling for the right choice.

In the above discussion, we only considered indefinite integrals. For the definite integral , we have two ways to go:

Evaluate the indefinite integral

which gives

2 Use the above steps describing Integration by Parts directly on the given definite integral. This is how it goes:

(i) Write down the given definite integral

where you identify the two functions f(x) and g(x).

(ii) Introduce the intermediary functions u(x) and v(x) as:

Then you need to make one derivative (of f(x)) and one integration (of g(x)) to get

(iii) Use the formula

(iv) Take care of the new integral

Example

Evaluate

Let us follow the steps

1 This is an indefinite integral involving one function. The second needed function is g(x) = 1. Since the derivative of this function is 0, the only choice left is to differentiate the other function .

2 We have

which gives

3 We have the formula

4 Since

we get

Since the derivative of

is 1/x, it is very common that whenever an integral with another function, to

involves a function which is a product of differentiate and integrate the other function.

1.2

Integration of Rational functions


A rational function is by definition the quotient of two polynomials. For example

are all rational functions. Remember in the definition of a rational function, you will not see neither or |x| for example. Note that integration by parts will

not be enough to help integrate a rational function. Therefore, a new technique is needed to do the job. This technique is called Decomposition of rational functions into a sum of partial fractions (in short Partial Fraction Decomposition).

Let us summarize the practical steps how to integrate the rational function

1 If go to step 2.

, perform polynomial long-division. Otherwise

2 Factor the denominator Q(x) into irreducible polynomials: linear and irreducible quadratic polynomials. 3 Find the partial fraction decomposition. 4 Integrate the result of step 3.

Remark: The main difficulty encountered in general when using this technique is in dealing with step 2 and step 3. Therefore, it is highly recommended to do a serious review of partial decomposition technique before adventuring into integrating fractional functions.

Example1

Find

Solution. Since the degree of the numerator is higher than the denominator, we should perform the long-division. We get

which implies

We concentrate on the fraction gives

. The partial decomposition technique

This gives x + 2 = A(x-1) + B(x+1). If we substitute x=1, we get B = 3/2 and we substitute x=-1, we get A = -1/2. Therefore, we have

Now we are in position to perform the desired integration. Indeed, we have

Since we have

we get

1.3

Substitution
Many integrals are hard to perform at first hand. A smart idea consists in ``cleaning'' them through an algebraic substitution which transforms the given integrals into easier ones. Let us first explain how the substitution technique works.

1 Write down the given integral

2 Come up with a substitution u = u(x). 3 Ideally you may want to find the inverse function of u(x), meaning that you will find x = x(u).

4 Differentiate to find dx = x'(u) du.

5 Back to the given integral and make the appropriate substitutions

6 Check after algebraic simplifications that the new integral is easier than the initial one. Otherwise, go back to step 2 and come up with another substitution.

7 Do not forget that the answer to

is a function of x. Therefore once

you have finished doing all your calculations, you should substitute back to the initial variable x.

Remarks. 1 In general, if the substitution is good, you may not need to do step 3. Indeed, from u= u(x), differentiate to find du=u'(x)dx. Then substitute the new variable

u into the integral

. You should make sure that the old variable x

has disappeared from the integral.

2 A better substitution is sometimes hard to find at first hand. Therefore we do not recommend spending a lot of time in step 2 trying to find it. After a while you may start to have a good feeling for the best substitution.

3 If you are given a definite integral

, nothing will change except in

step 5 you will have to replace a and b also, that is

In this case, you will never have to go back to the initial variable x.

1.4

Trigonometric substitutions
The familiar trigonometric identities

may be used to eliminate radicals from integrals. Specially when these integrals involve and .

1 For

set

. In this case we talk about sine-substitution.

2 For substitution.

set

. In this case we talk about tangent-

3 For substitution.

set

. In this case we talk about secant-

The expressions

and

should be seen as a constant plus-minus a

square of a function. In this case, x represents a function and a a constant. For example can be seen as one of the two previous expressions.

Indeed, if we complete the square we get

where

So from the above substitutions, we will set

Example 1

Find

Solution. It is easy to see that sine-substitution is the one to use. Set or equivalently . Then which

gives us

Easy calculations give

Technique of integration of powers of trigonometric functions give

which suggests the substitution implies

. Hence

which

Therefore, we have

This will not answer fully the problem because the answer should be given as a

function of x. Since after easy simplifications

, we get

1.5

Rational Expressions of Trigonometric Functions


Expressions like

are called rational expressions of sin and cos. Note that all the other trigonometric functions are rational functions of sin and cos. The main idea behind integrating such functions is the general substitution

In order to have better feeling how things do work, remember the trigonometric formulas

It is not hard to generate similar formulas for

, and will

from the above formulas. Therefore, any rational function

be transformed into a rational function of t via the above formulas. For example, we have

where

. Note that in order to complete the substitution we need to , we get

find dx as function of t and dt. Since

Now we are ready to integrate rational functions of sin and cos or at least transform them into integrating rational functions.

Example Find

Solution. Set

, then we have

This clearly implies

Algebraic simplifications give

The technique of partial decomposition gives

This will not complete our answer since we need to go back the variable x. Indeed, we have

1.6

Summary
Many integrals are hard to perform at first hand. A smart idea consists in ``cleaning'' them through an algebraic substitution which transforms the given integrals into easier ones.

1.7

Keywords
Trigonometric substitutions

The familiar trigonometric identities

may be used to eliminate radicals from integrals. Specially when these integrals involve and .

1.8

Self Assessment Questions


1. Explain the techniques of integration? 2. What is trigonometric substitution? 3. Discuss Integration of Rational functions. 4. Explain Trigonometric substitutions. 5. Discuss Substitution. 6. Explain Rational Expressions of Trigonometric Functions.

1.9

References
Elementary Engineering Mathematics by Dr. B.S. Grewal, Khanna Publications Higher Engineering Mathematics by B.S. Grewal, Khanna Publications Differential Calculus by Shanti Narayan, Publishers S. Chand & Co. Integral Calculus by Shanti Narayan, Publishers S. Chand & Co.

Modern Abstract Algebra by Shanti Narayan, Publishers S. Chand & Co. The Differential and Integral Calculus Part 2 Augustus De Morgan (Author) Publisher: Kessinger Publishing, LLC; illustrated edition edition (April 5, 2004) Integral Calculus for Competetion, Shanti Narayan, Paperback,S. Chand Group

Unit 2 Integration of Trigonometric Functions


Structure 2.0 2.1 2.2 2.3 2.4 2.5 Objectives Introduction Product of Sines and Cosines Trigonometric Powers Integration of Nonelementary Functions Local behaviour of functions 2.5.1 Taylor Polynomials 2.5.2 Indeterminate Forms 2.5.3 Indeterminate Quotient Forms

2.5.3.1 Indeterminate Quotient Form 2.6 2.7 2.8 2.9 Summary Keywords Self Assessment Questions References

2.0

Objectives
After studying this unit you will be able to: Define Reduction Formula Explain Product of Sines and Cosines Define Trigonometric Powers Describe Integration of Nonelementary Functions Explain Local behaviour of functions Discuss Taylor Polynomials Describe Indeterminate Forms

2.1

Introduction
Let us understand that some integrals of the following form are :

We have two cases: both m and n are even or at least one of them is odd. Case I: m or n odd

Suppose n is odd. Hence n = 2k + 1. So

hold.

Therefore, we have

which suggests the substitution and hence

. Indeed, we have

The latest integral is a polynomial function of u which is easy to integrate.

Remark. Note that if m is odd, then we will split calculations. In this case, the substitution will be

and carry the same .

Example 1 Case II: m and n are even

The main idea behind is to use the trigonometric identities

Example 2 Remark. The following two formulas may be helpful in integrating powers of sine and cosine.

2.2

Product of Sines and Cosines

Here we will discuss the following integrals

The case m=n is very easy to handle. Therefore we will consider only the case . We will need the following trigonometric identities

Example: Find

Solution. We have the trigonometric identity

Therefore, we get

which yields

2.3

Trigonometric Powers
These are integrals of the form

In every single one of these integrals, we will develop what is commonly called a Reduction Formula. The main idea behind is a smart use of trigonometric identities. Let us describe how it works.

For

, set

For

, set

For

, set

For

, set

Let us show how one can generate a reduction formula for once, will be given without any proof. We have

. The other

Since the derivative of

is

, we get

Therefore, we have

This is the reduction formula associated to the tangent function. What it says is that in order to find the integral of it is enough to find the integral of

. This way, we can reduce the power n all the way down to 1 or 0.

Recall that reduction formulas.

. Let us give a table for all the

where a is an arbitrary constant and

Example 1

Remark. Note that for

and

when n is even can be

handled in a easier way. Indeed, we have

which suggests the substitution

. The same idea works for the ).

cosecant function (in this case, the substitution will be

2.4

Integration of Nonelementary Functions


This page was motivated by many discussions on the cyberboard regarding

e-x2dx

Liouville first proved (in 1835) that if f (x) and g(x) are rational functions (where g(x) is not a constant), then f (x)eg(x)dx is elementary if and only if there exists a

rational function R(x) such that

f (x) = R'(x) + R(x)g(x).

You should be able to use this theorem to easily show that elementary.

e-x2dx is NOT

This theorem can also be used to prove that integrals like

xxdx,

dx,

dx,

dx

are NOT elementary.

Alternatively, if

f (x)eg(x)dx can be integrated in a finite number of terms using

elementary functions, then the primitive must be R(x)eg(x) for some rational function R(x).

So by taking the derivatives of both sides of

f (x)e-x2dx = R(x)e-x2

and matching powers, you can show that number of elementary terms.

e-x2dx cannot be integrated in a finite

Additionally, Chebyshev first proved that

xp(a + bxr)qdx can be integrated in a

finite number of elementary terms if and only if at least one of

, q or

+ q is an integer.

2.5

Local behaviour of functions


2.5.1 Taylor Polynomials The fundamental idea in differential calculus is that a function can be ``locally'' approximated by its tangent line.

For instance consider the function equals as

near , the tangent line at

. Since its derivative at can be written

In the picture below, the sine function is black, while its tangent line is depicted in red. Close to , both are quite close!

2.5.2 Indeterminate Forms When we deal with the limits of quantities, very often we have to compare numbers such as: adding two large numbers; multiplying two large numbers; subtracting two large numbers; multiplying a large number with a small number

close to 0, will be called a small number, while a number close to called a large number. Note that

will be

is a very large number which happens to be is the

negative. Many are confused about this point since they believe that

smallest "number" among the real numbers. Again large and small here is to be understood in terms of quantities while the set of real numbers has a natural order which is not of concerns to us here.

Let us give some indeterminate forms which we will take care of in the next pages:

or

Remember that the inverse of a small number is a large number while the inverse of a large number is a small number, that is

Remark. The inverse of a small number is a large one, this is true sizewise but we do have to work little harder to find out about the sign of the large number ( is a positive large number while is a negative large number). This is the

only time when we have to find out about the 0 whether it is positive or negative so we can say something about it inverse. We will write 0+ to designate a positive small number while 0- will designate a negative small number.

2.5.3 Indeterminate Quotient Forms

2.5.3.1 Indeterminate Quotient Form May be the most natural indeterminate form is the quotient of two small numbers

or

. Equivalently another natural indeterminate form is the quotient of two large

numbers or

. In both cases, it is very easy to convince oneself that nothing

can be said, in other words we have no conclusion. It is very common to see

students claiming case.

. We hope this page will convince some that it is not the

Hpital's Rule: Though this rule was named after Hpital, it is Bernoulli who did discover it in the early 1690s. This rule answers partially the problem stated above. Indeed, let f(x) and g(x) be two functions defined around the point a such that

Then we have

Next we take the ratio function limit. Hpital's rule states that if

. Do any needed algebra and then find its

then we have

Remark. Note that if

then you can use Hpital's rule for the ratio function

, by looking for

In other words, there is no limit where to stop.

Example. Find the limit

Answer. We have

. Hence

Clearly we are in full swing to use Hpital's rule. We have

Therefore we have

Example. Fint the limit

Answer. We have

Hence we can use Hpital's rule. Since , we have

and

So it is clear that we need to use Hpital's rule another time. But since we proved in the example above

we conclude that

Therefore, we have

Remark. The above examples have a wonderful implication. Indeed, the first example implies that when that when then then . . The second example implies

Example. Fint the limit

Answer. Set

and

. We have f(0) =

g(0) = 0. So we have all assumptions satisfied to use Hpital's rule. We have

Clearly we have

So

we

use

Hpital's

rule

again.

Set

and

. Then we have

Again we have

In fact another use of Hpital's rule makes the functions involved even more complicated. So what do we do in this case? A partial answer is given but the use of Taylor Polynomials.

Taylor Polynomial's Technique. First recall the assumptions of the original problem: let f(x) and g(x) be two functions defined around the point a such that

Using Taylor Polynomials, we get around a (that is

and

where n and m are natural numbers. Since f(a) = g(a) =0, we get

and

But we may have the next derivatives also equal to 0 at a. Hence we are sure that there exist two natural numbers N and M such that

and

when

. This clearly implies

So the job is over. Indeed, it is now clear that the limit

is not a problem and depends on the natural numbers N and M.

Before we do any example showing the power behind this technique, recall that one may use all the properties of Taylor Polynomials.

Example. Fint the limit

Answer. First we consider the basic functions which generate the functions involved in this limit, that is Polynomials of these functions and . Next we write the Taylor

and

Note that if more terms are needed, we will come back and put the next terms. Using properties of Taylor Polynomials, we get

and

Hence we have

One should appreciate the beauty and power behind this technique in comparing the above calculations with the ones done under Hpital's rule.

2.6

Summary
Liouville first proved (in 1835) that if f (x) and g(x) are rational functions (where g(x) is not a constant), then f (x)eg(x)dx is elementary if and only if there exists a

rational function R(x) such that

f (x) = R'(x) + R(x)g(x).

2.7

Keywords
Reduction Formula:

This is the reduction formula associated to the tangent function.

2.8

Self Assessment Questions


1. What is Reduction Formula? 2. Explain Integration of Nonelementary Functions. 3. Explain Product of Sines and Cosines. 4. Define Trigonometric Powers . 5. Describe Integration of Nonelementary Functions. 6. Explain Local behaviour of functions. 7. Discuss Taylor Polynomials. 8. Describe Indeterminate Forms.

2.9

References
Elementary Engineering Mathematics by Dr. B.S. Grewal, Khanna Publications Higher Engineering Mathematics by B.S. Grewal, Khanna Publications Differential Calculus by Shanti Narayan, Publishers S. Chand & Co. Integral Calculus by Shanti Narayan, Publishers S. Chand & Co. Modern Abstract Algebra by Shanti Narayan, Publishers S. Chand & Co. The Differential and Integral Calculus Part 2 Augustus De Morgan (Author) Publisher: Kessinger Publishing, LLC; illustrated edition edition (April 5, 2004)

Integral Calculus for Competetion, Shanti Narayan, Paperback,S. Chand Group

Unit 3 Partial Fractions


Structure

3.0 3.1 3.2 3.3 3.4 3.5

Objectives Introduction Polynomial Long Division Summary Keywords Self Assessment Questions

3.0

Objectives
After studying this unit you will be able to: Define Polynomial Long Division Discuss Long Polynomial Division and Factoring

3.1

Introduction
Let us understand that Early in Algebra you learn how to combine "simple'' fractions into a "more complicated'' one. Here is a typical example:

The Method of Partial Fractions does the opposite: It dissects a complicated fraction into a sum of simple fractions. While this is a little more complicated than going the other direction, it is also more useful. Major applications of the method of partial fractions include: Integrating rational functions in Calculus Finding the Inverse Laplace Transform in the theory of differential equations

How simple can it get?

A simple fraction is a fraction with a simple denominator. The first step consists of detecting the factors (the building blocks) of the given denominator. The Fundamental Theorem of Algebra tells us what is possible: Every polynomial can be factored into linear factors (degree 1 polynomials) and irreducible polynomials of degree 2.

Some Examples.

The polynomial

can be factored into three linear factors as follows

(Do you remember how to do this?):

The polynomial

has a linear factor and an irreducible factor of degree 2:

The polynomial 2:

has a twice repeated irreducible factor of degree

How can you tell whether a degree 2 polynomial is irreducible (over the field of real numbers), or can be factored further into two linear factors?

There are different methods to decide:

Graphically: A reducible quadratic polynomial has 2 zeros or one repeated zero, an irreducible quadratic polynomial has no zeros!

This quadratic polynomial can be factored

An irreducible quadratic polynomial

Algebraically: If the quadratic formula results in a negative expression under the radical (the discriminant), the associated polynomial is irreducible: Consider the polynomial yields: : Using the quadratic formula for

Since the discriminant (the expression under the radical) is negative, the polynomial is irreducible!

3.2

Polynomial Long Division


An Example.

In this section you will learn how to rewrite a rational function such as

in the form

The expression

is called the quotient, the expression

is called the divisor and the term

is called the remainder. What is special about the way the expression above is written? The remainder 28x+30 has degree 1, and is thus less than the degree of the divisor .

It is always possible to rewrite a rational function in this manner:

DIVISION ALGORITHM: If f(x) and

are polynomials, and

the degree of d(x) is less than or equal to the degree of f(x), then there exist unique polynomials q(x) and r(x), so that

and so that the degree of r(x) is less than the degree of d(x). In the special case where r(x)=0, we say that d(x) divides evenly into f(x).

How do you do this? Let's look at our example

in more detail. Write the expression in a form reminiscent of long division:

First divide the leading term

of the numerator polynomial by the leading term

of the divisor, and write the answer 3x on the top line:

Now multiply this term 3x by the divisor

, and write the answer

under the numerator polynomial, lining up terms of equal degree:

Next subtract the last line from the line above it:

Now repeat the procedure: Divide the leading term the last line by the leading term to the 3x on the top line:

of the polynomial on

of the divisor to obtain -11, and add this term

Then multiply "back":

and write the

answer under the last line polynomial, lining up terms of equal degree:

Subtract the last line from the line above it:

You are done! (In the next step, you would divide 28x by

, not yielding a

polynomial expression!) The remainder is the last line: 28x+30, and the quotient is the expression on the very top: 3x-11. Consequently,

How to check your answer?

The easiest way to check your answer algebraically is to multiply both sides by the divisor:

then to multiply out:

and then to simplify the right side:

Indeed, both sides are equal! Other ways of checking include graphing both sides (if you have a graphing calculator), or plugging in a few numbers on both sides (this is not always 100% foolproof).

Another Example. Let's use polynomial long division to rewrite

Write the expression in a form reminiscent of long division:

First divide the leading term

of the numerator polynomial by the leading term x on the top line:

of the divisor, and write the answer

Now multiply this term

by the divisor x+2, and write the answer

under the numerator polynomial, carefully lining up terms of equal degree:

Next subtract the last line from the line above it:

Now repeat the procedure: Divide the leading term

of the polynomial on the

last line by the leading term x of the divisor to obtain -2x, and add this term to the on the top line:

Then multiply "back":

and write the answer under

the last line polynomial, lining up terms of equal degree:

Subtract the last line from the line above it:

You have to repeat the procedure one more time.

Divide:

Multiply "back":

and subtract:

You are done! (In the next step, you would divide -9 by x, not yielding a polynomial expression!) The remainder is the last line: -9 (of degree 0), and the quotient is the expression on the very top: . Consequently,

An Example: Long Polynomial Division and Factoring.

Let's use polynomial long division to rewrite

Write the expression in a form reminiscent of long division:

First divide the leading term

of the numerator polynomial by the leading term of

the divisor, and write the answer x on the top line:

Now multiply this term x by the divisor

, and write the answer

under the numerator polynomial, carefully lining up terms of equal degree:

Next subtract the last line from the line above it:

Now repeat the procedure: Divide the leading term last line by the leading term x on the top line:

of the polynomial on the

of the divisor to obtain -5, and add this term to the

Then multiply "back":

and write the answer under

the last line polynomial, lining up terms of equal degree:

Subtract the last line from the line above it:

You are done! In this case, the remainder is 0, so .

divides evenly into

Consequently,

Multiplying both sides by the divisor

yields:

In this case, we have factored the polynomial have written it as a product of two "easier" (=lower degree) polynomials.

, i.e., we

Let us summarize the practical steps how to integrate the rational function

1 If go to step 2.

, perform polynomial long-division. Otherwise

2 Factor the denominator Q(x) into irreducible polynomials: linear and irreducible quadratic polynomials. 3 Find the partial fraction decomposition. 4 Integrate the result of step 3.

Remark: The main difficulty encountered in general when using this technique is in dealing with step 2 and step 3. Therefore, it is highly recommended to do a serious review of partial decomposition technique before adventuring into integrating fractional functions.

3.3

Summary
A simple fraction is a fraction with a simple denominator. The first step consists of detecting the factors (the building blocks) of the given denominator. The Fundamental Theorem of Algebra tells us what is possible:

3.4

Keywords
Method of Partial Fractions: The Method of Partial Fractions dissects a complicated fraction into a sum of simple fractions. While this is a little more complicated than going the other direction, it is also more useful.

3.5

Self Assessment Questions


1. What is Polynomial Long Division?

2. Explain Long Polynomial Division and Factoring.

3.6

References
Elementary Engineering Mathematics by Dr. B.S. Grewal, Khanna Publications Higher Engineering Mathematics by B.S. Grewal, Khanna Publications Differential Calculus by Shanti Narayan, Publishers S. Chand & Co. Integral Calculus by Shanti Narayan, Publishers S. Chand & Co. Modern Abstract Algebra by Shanti Narayan, Publishers S. Chand & Co. The Differential and Integral Calculus Part 2 Augustus De Morgan (Author) Publisher: Kessinger Publishing, LLC; illustrated edition edition (April 5, 2004) Integral Calculus for Competetion, Shanti Narayan, Paperback,S. Chand Group

Unit 4 Improper Integrals


Structure

4.0 4.1 4.2 4.3 4.4 4.5 4.6 4.7

Objective Introduction Convergence and Divergence of Improper Integrals Tests of Convergence Summary Keywords Self Assessment Questions References

4.0

Objective
After studying this unit you will be able to: Define Improper Integrals Discuss Convergence and Divergence of Improper Integrals Explain Tests of Convergence

4.1

Introduction

Let us understand that the definition of an integral

requires the

function f(x) to be bounded on the bounded interval [a,b] (where a and b are two real numbers). It is natural then to wonder what happens to this definition if

1 the function f(x) becomes unbounded (we call this case Type I);

2 the interval [a,b] becomes unbounded (that is case Type II).

or

)(we call this

In both cases, we say that the integral

is improper.

Case Type I: Consider the function f(x) defined on the interval [a,b] (where a and b are real numbers). We have two cases f(x) becomes unbounded around a or unbounded around b (see the images below)

and

For the sake of illustration, we considered a positive function. The integral

represents the area of the region bounded by the graph of f(x), the xaxis and the lines x=a and x=b. Assume f(x) is unbounded at a. Then the trick behind evaluating the area is to compute the area of the region bounded by the graph of f(x), the x-axis and the lines x=c and x=b. Then we let c get closer and closer to a (check the figure below)

Hence we have

Note that the integral

is well defined. In other words, it is not an

improper integral. If the function is unbounded at b, then we will have

Remark. What happened if the function f(x) is unbounded at more than one point on the interval [a,b]?? Very easy, first you need to study f(x) on [a,b] and find out where the function is unbounded. Let us say that f(x) is unbounded at example, with and (that is . Then you must choose a number ) and then write and between for

Then you must evaluate every single integral to obtain the integral

Note that the single integrals do not present a bad behavior other than at the end points (and not for both of them).

Example. Consider the function

defined on [0,1]. It is easy to

see that f(x) is unbounded at x = 0 and integral

. Therefore, in order to study the

we will write

and then study every single integral alone.

Case Type II: Consider the function f(x) defined on the interval

or

. In other words, the domain is unbounded not the function (see the figures below).

and The same as for the Type I, we considered a positive function just for the sake of illustrating what we are doing. The following picture gives a clear idea about what we will do (using the area approach)

So we have

and

Example. Consider the function

defined on

. We have

On the other hand, we have

Hence we have

It may happen that the function f(x) may have Type I and Type II behaviors at the same time. For example, the integral is one of them. As we did before, we must always split the integral into a sum of integrals with one improper behavior (whether Type I or Type II) at the end points. So for example, we have

The number 1 may be replaced by any number between 0 and

since the

function behavior at .

has a Type I behavior at 0 only and of course a Type II

4.2

Convergence and Divergence of Improper Integrals


Consider a function f(x) which exhibits a Type I or Type II behavior on the interval

[a,b] (in other words, the integral

is improper). We saw before that

the this integral is defined as a limit. Therefore we have two cases:

1 the limit exists (and is a number), in this case we say that the improper integral is convergent; 2 the limit does not exist or it is infinite, then we say that the improper integral is divergent.

If the improper integral

is split into a sum of improper integrals

(because f(x) presents more than one improper behavior on [a,b]), then the integral converges if and only if any single improper integral is convergent. Example. Consider the function on [0,1]. We have

Therefore the improper integral

converges if and only if the improper integrals

are convergent. In other words, if one of these integrals is divergent, the integral

will be divergent.

The p-integrals Consider the function

(where p > 0) for

. Looking at this function closely we see that f(x) presents an improper behavior at 0 and divergence of only. In order to discuss convergence or

we need to study the two improper integrals

We have

and

For both limits, we need to evaluate the indefinite integral

We have two cases:

if p=1, then we have

if

, then we have

In order to decide on convergence or divergence of the above two improper integrals, we need to consider the cases: p<1, p=1 and p >1.

If p <1, then we have

and

If p=1, then we have

and

If p > 1, we have

and

The p-Test: Regardless of the value of the number p, the improper integral

is always divergent. Moreover, we have

is convergent if and only if p <1

is convergent if and only if p >1

4.3

Tests of Convergence
It is very easy to see that a simple improper integral may be very hard to decide whether it is convergent or divergent. For example, the improper integral

is hard to study since it is very difficult to find an antiderivative of the function

. The tests of convergence are very useful tools in handling such improper integrals. Unfortunately some improper integrals fails to fall under the scope of these tests but we will not deal with them here.

Recall the p-Test: Regardless of the value of the number p, the improper integral

is always divergent. Moreover, we have

is convergent if and only if p <1

is convergent if and only if p >1

Note that one may generalize this test to include the following improper integrals

The conclusion is similar to the above one. Indeed we have

is convergent if and only if p <1

Comparison Test Let f(x) and g(x) be two functions defined on [a,b] such that

for any

. Then we have

If

is convergent, then

is convergent.

If

is divergent, then

is divergent.

Example. Decide on the convergence or divergence of

Answer. We have for

The p-Test implies that the improper integral the Comparison test implies that the improper integral

is convergent. Hence

is convergent.

We should appreciate the beauty of these tests. Without them it would have been almost impossible to decide on the convergence of this integral.

Before we get into the limit test, we need to recall the following: we will say and write when if and only if

Limit test Let f(x) and g(x) be two positive functions defined on [a,b]. Assume that both functions exhibit an improper behavior at a and , then we when have

is convergent if and only if

is convergent.

This statement is still valid whether a is a finite number or infinite or if the improper behavior is at b.

Example. Establish the convergence or divergence of

Answer. Clearly this integral is improper since the domain is unbounded (Type

II). Moreover since the function

is unbounded at 0, then we also have

an improper behavior at 0. First we must split the integral and write

First let us take care of the integral

. Since

when

, and (because of the p-test) the integral

is convergent, we deduce from the limit test that

is convergent. Next we investigate the integral

. Since

when

, and (because of the p-test) the integral

is convergent, we deduce from the limit test that

is convergent.

Remark. One may notice that in the above example, we only used the limit test combined with the p-test. But we should keep in mind that it is not the case in general. The next example shows how the use of other tests is more than useful.

Example. Establish the convergence or divergence of

Answer. Again it is easy to see that we have an improper behavior at both 0 and . Hence we must split the integral and write

The integral

is easy to take care of since we have

and because implies that

is convergent (by the p-test), the basic comparison test

is convergent. Next we take care of the integral limit test. Indeed, since when

. Here we use the , then we have

Because integral

is divergent (by the p-test), then the limit test implies that the

is divergent. Conclusion the improper integral

is divergent.

Remark. One may argue that the above example is in fact not a good one to illustrate the use of different tests. Since if we have showed first that the integral

is divergent via the limit test, then we do not need to take care of the other integral and conclude to the divergence of the given integral. A very good point. Now consider the improper integral

and show that in this case the integral is convergent. Let us point out that the trigonometric functions are very bad when it comes to look at what is happening at . Hence the limit test is absolutely not appropriate to use...

Example. Establish the convergence or divergence of

Answer. This is clearly not an improper integral of Type II. Let us check if it is of Type I. First notice that . Hence the function is

unbounded at x=1 and x=3 (you must check it by taking the limit.. left as an exercise). Since 3 is between 2 and 4, we deduce that the integral is improper and the only bad point is 3. Hence we must split the integral to get

Let us take care of the integral . It is easy to see that when

, then we have

The p-test implies that the integral

is convergent. Hence by the limit test we conclude that the integral

is convergent. Using the same arguments, we can show that the integral

is also convergent. Therefore the integral

is convergent.

4.4

Summary
It is very easy to see that a simple improper integral may be very hard to decide whether it is convergent or divergent. For example, the improper integral

is hard to study since it is very difficult to find an antiderivative of the function

4.5

Keywords
Comparison Test Let f(x) and g(x) be two functions defined on [a,b] such that

for any

. Then we have

If

is convergent, then

is convergent.

If

is divergent, then

is divergent.

4.6

Self Assessment Questions


1. Define Improper Integral? 2. Explain Convergence and Divergence of Improper Integrals.

4.7

References
The Differential and Integral Calculus Part 2 Augustus De Morgan (Author) Publisher: Kessinger Publishing, LLC; illustrated edition edition (April 5, 2004) Integral Calculus for Competetion, Shanti Narayan, Paperback,S. Chand Group

Elementary Engineering Mathematics by Dr. B.S. Grewal, Khanna Publications Higher Engineering Mathematics by B.S. Grewal, Khanna Publications Differential Calculus by Shanti Narayan, Publishers S. Chand & Co. Integral Calculus by Shanti Narayan, Publishers S. Chand & Co. Modern Abstract Algebra by Shanti Narayan, Publishers S. Chand & Co.

You might also like